MBE Mid-Term wrong answers

Pataasin ang iyong marka sa homework at exams ngayon gamit ang Quizwiz!

QID: 9280 A hunting club owns and operates a hunting ranch abutting a national forest. Club members hunt elk, deer, and big-horned sheep on the premises of the 2,000 acre ranch and fish for bass on the ranch's man-make lake. Pursuant to a federally funded wildlife rehabilitation program, the U.S. Forest Service reintroduced mountain lions to the national forest. Mountain lions had previously lived in the area, but had been hunted to extinction. The lions introduced into the environment were from a breeding program in another part of the country. The animals soon bred so successfully in the wild that their hunting territories began to spread throughout the state. Human hunters began to complain that the mountain lions were decimating the hers of elk, deer, and sheel and significantly reducing the number hoofed prey available for slaughter for meat or trophies. There was no statistical proof of these predations by mountain lions, but there was anecdotal evidence that herds of game animals were suffering greater casualties. The hunting club filed suit in federal court to prohibit the Forest Service form further introduction of mountain lions and to require strict controls on the number of lions per hunting territory in the region. The U.S> Forest Service filed a motion to dismiss the suit. How is the court likely to rule on the motion to dismiss? (A) The court will deny the motion, because the hunting club has met the threshold of showing a case or controversy (B) The court will grant the motion, because the case is not ripe. (C) The court will grant the motion, because the hunting club has not suffered a direct injury. (D) The court will grant the motion and dismiss the case, because it presents a political question.

(A). A case is not ripe for review unless an actual controversy exists--that is, unless the plaintiff has experienced an actual injury or a threat of imminent injury. In this case, even if there is no statistical proof that herds of elk, deer, and sheep are suffering due to mountain lions, there is sufficient anecdotal evidence that hers are being negatively affected. There is, therefore an imminent threat of injury to members of the rod and gun club who hunt the herds for meat and trophies. Thus, the hunting club's claim meets the "case or controversy" requirement of Article III of the U.S. Constitution. Even if the hunting club as an association has not suffered a direct injury, its members have suffered a direct injury or imminent threat of injury, and the club can assert association standing on their behalf.

QID: 101191 At a federal jury trial involving a serious highway accident, the defendant presented ample evidence, including expert witness testimony, forensic evidence, and information from her car's event data recorder showing that she had tried to avoid the crash. The plaintiff presented little evidence, but made a strong emotional appeal based on the severity of her injuries. The jury awarded the $400,000 that the plaintiff had sought in various types of compensation, but added $400,000 in punitive damages that the plaintiff had not sought. The defendant did not believe that she should have been found liable, so she moved for a new trial. Should the court grant the motion for a new trial? (A) Yes, because the jury verdict is so inappropriate that the jury seems to have acted with extreme prejudice. (B) Yes, because a reasonable jury could not have interpreted the evidence in a way that would support this verdict. (C), No, because the court must view the evidence in the light most favorable to the verdict. (D) No, because there is no evidence that the judge committed reversible error.

(A). A new trial can be granted on all or some of the issues presented at trial and is appropriate in several situations, including where a jury has acted in a way that can only be explained by its having misunderstood its task or having been seriously prejudiced against a party. Here, the jury awarded substantial punitive damages in what appears to be a negligence case where it is entirely possible that there should not even have been a defense verdict. As a result, a new trial award is appropriate. Form the information presented, it does appear that the jury operated from complete misunderstanding of its task or strong prejudice against the defendant. However, (B) states the standard for judgment as a matter of law, not for a new trial award.

QID: 9171 A defendant is arrested and charged with the arson of a schoolhouse. At trial, the prosecution offers the testimony of the defendant's neighbor that the day after the arson, when she noticed that the defendant had not been home the night before, she said: "Where were you last night, lighting up the schoolhouse?" The defendant did not respond. Is this testimony admissible? (A) No, because the defendant had no reason to respond to his neighbor's comment. (B) No, because admitting it would violate the defendant's constitutional rights. (C) Yes, as a statement by an opposing party. (D) Yes, as a statement against interest.

(A). A statement by a party-opponent is an out-of-court declaration consisting of words or conduct that is made or adopted by a party (or others) and offered against that party. The Federal Rules of Evidence classify statements by a party-opponent as nonhearsay. Under certain circumstances, a party's failure to respond (by statement or act) in the face of a provocative accusation or event may be classified as an admission by silence. An admission by silence considers the failure by a party, in whose presense, hearing, or observation of an act or declaration is made, to assert that such act or declaration is untrue. Admission by silence is based upon the principle that when the act or declaration is such as naturally to call for action or comment if not true, the party against whom such act or declaration is made must assert it as untrue if it is proper and possible for him/her to do so. The party's failure to respond will be an admission if: (1) the party against whom the evidence is offered heard, understood, and was capable of responding to the statement; and (2) a reasonable person in the party's position would have responded (e.g., denied the accusation). Here, even assuming that the defendant heard and understood his neighbor's question, a reasonable person in his position would likely have interpreted it as a joke, and thus not requiring a denial. Under these circumstances, the neighbor's question and the defendant's silence cannot be taken as an admission by the defendant. As such, the neighbor's question and the defendant's failure to respond are inadmissible.

QID: 9247 A confidential informant tells a police detective that a business owner has an active gambling and loan-sharking business. The informant, from whom the detective has previously received reliable information, says that he has firsthand information that the business owner launders money from the illegal activities through his restaurant. The informant also states that the business owner keeps outstanding loan and betting records at his home and that every Wednesday afternoon he moves the settled loan and betting records in his car to a safe deposit box. The detective sets up serveillance outside of the busienss owner's house. Over the course of three weeks, he sees heavy traffic going in and out of the business owner's house, particularly on Friday and Saturday, the days before college and professional football games. The detective then goes on a two-week vacation. Upon his return, he applies for an obtains a search warrant for gambling and loan-sharking records at the business owner's house. After a thorough search of the busienss owner's house, police find the records, which date back several years. The detective immediately calls a magistrate and swears out an arrest warrant for the busienss owner, which they execute at the business owner's place of business. The evidence discovered at the business owner's home will most likely be (A) admitted, because probable cause existed to support issuance of the search warrant. (B) admitted, because the search was conducted incident to a lawful arrest. (C) suppressed, because the warrant was based at least partially on hearsay. (D) suppressed, because the information that the warrant was based on was stale as a result of the detective's vacation.

(A). A valid search warrant must be issued by a neutral and detached magistrate who reviews evdience submitted by law enforcement and determines that probable cause exists. Probable cause exists when the information presented to the magistrate is sufficient that a reasonable person would concluded that it was more probable than not that evidence of named items or persons would be found. The warrant may be issued based on an informant's tip where probable cause is established through the totality of the circumstances. Two of the factors in assessing the totality of the circumstances are the reliaiblity of the informant and police corroboration. In addition, a warrant must "describe with particularity" the place to be searched and the items or persons to be seized. In this case, the informant has shown himself to be credible in the past, and the detective's observations supported the informant's claims. Because the informant was considered reliable, and the tip had been corroborated by the police, sufficient probable cause existed to support a search warrant being issued. It is true that information used to support issuance of a search warrant must not be outdated. However, given the type of operation in progress, the information from the informant indicating that the illegal activities had been ongoing over time, and the fact that law enforcement had observed the operation over a period of weeks, it is unlikely that the information would be deemed "no longer relevant" simply because of a two-week delay.

QID: 9260 A woman inherits a parrot form her father. Not knowing anything about parrots, she puts the parrot up for sale for $50, describing it as a male parrot. A student purchases the bird from the woman as a gift for his mother, who specifically wanted a male bird. A subsequent veterinary examination reveals that the parrot is actually a female and is worth $4,500. When the woman learns the truth, she files a lawsuit seeking the return of the bird from the student. Will the woman prevail? (A) Yes, because the parties made a mutual mistake. (B) Yes, because the law does not aid inequity. (C) No, because $50 was adequate consideration for the contract. (D) No, because the law holds an offeror responsible for her own mistake.

(A). While a mutual mistake as tot he value of an item sold does not negate the validity of a contract, a mutual mistake going to the "heart of the agreement" does. Here, the woman advertised the bird as male, and the student specifically sought a male bird; thus the sex of the bird was a crucial element of the contract. Given the parties' mutual mistake as to this essential element of the contract, the court will likely hold that no enforceable contract existed and rule in the woman's favor.

QID: 9124 A woman is on trial for murder. The prosecution calls a police detective to testify against her. On cross-examination, the woman's attorney asks the detective to confirm that the murder victim, shortly before he died, stated aloud that the woman was not the one who killed him, and the detective answers that he is unable to recall whether the victim made such a statement. The woman's attorney seeks to refresh the detective's memory by asking him to read a case report prepared by another police officer immediately following the woman's arrest. Is it permissible for the woman's attorney to request that the detective read the police report? (A) Yes, but only to refresh the detective's recollection. (B) yes, but only if the report constitutes a business record. (C) No, because the report is hearsay not within any exception. (D) No, because the detective did not prepare or adopt the report.

(A). At trial, a witness may use a writing (or any other item) to refresh her memory for the purpose of testifying. However, the witness may not read from the writing while testifying, and the writing cannot be entered into evidence unless the adverse party seeks to inspect the writing and have the portions relating to the witness's testimony entered into the record. Because the report is to be used merely to refresh the detective's recollection, it is not required that the detective himself have prepared the report or adopted it after it was prepared. This answer confuses the rule for refreshing a witness's recollection with the "past recolleciton recorded" exception to the hearsay rule.

QID: 9123 A father took out a life insurance policy that provided that the proceeds of the policy would not be paid to his daughter, the beneficiary, until she reached the legal age of 18. The father did not name a contingent beneficiary. Three months after he bought the policy, the father was killed in a hiking accident. The daughter died a few weeks later at the age of 17, from injuries she sustained while trying to aid her father following his accident. How would a court most likely construe the clause regarding payment of the proceeds to the daughter? (A) The only legal significance of the daughter's attainment of the age of 18 is in regard to the time of payment of the proceeds. (B) The daughter's attainment of the age of 18 is a constructive condition concurrent (C) There is no legal significance in the daughter attainment of the age of 18 (D) The daughter's attainment of the age of 18 is a condition precedent to the insurance company's obligation to pay out any proceeds.

(A). Contracts may include a clause for payment such as the one providing that the daughter will not receive any of the policy proceeds until she reaches the age of 18. However, courts do not always see fit to interpret such clauses literally, but often view them simply as guidelines as to when payment is to occur. In this case, if the court literally interpreted the clause and enforced it as such, this action could result in a forfeiture. Because courts do not favor forfeitures, a court would most likely interpret the clause as one that sets a time for payment, rather than as a precondition that must be met before the daughter, or her estate, may receive the proceeds.

QID: 10399 A honeymooning couple was spending a week at a lake resort. It was late summer and most tourists had headed home but the water in the lake was still warm and the couple enjoyed the relative privacy. One day, the couple went down to the lake and walked to the far end of the resort's beach for some privacy. The couple took off their clothes and went for a dip in the lake. When they spotted some people on the beach, they remained quiet, not wanting to attract attention to themselves. The people left and the couple decided to get dressed. Unfortunately, their clothes had been taken, presumably by the people they had seen. They retreated back into the water for awhile, unsure of how to proceed. Finally, the husband dared to look up and down the beach for a discarded towel or other means of cover. He found two long t-shirts on a chair. Using the shirts to cover up as best they could, the couple made their way back to their room. After getting dressed, the couple returned to the beach to look for their clothes. The husband's cell phone was in the pocket of his shorts, along with his wallet. They found two security guards chuckling near an outdoor shower and there were all of their clothes. It was obvious that the security guards had known that the couple was in the lake when the took the couple's clothing. Are the security guards guilty of false imprisonment? (A) Yes, because the couple believed that they were confined to the water. (B) Yes, because the security guards intentionally took the clothing, even if they did not intend to confine the couple. (C) No, because the couple found a means to escape. (D) No, because the couple was not physically confined.

(A). False imprisonment requires that the perpetrators intentionally confined the victims and that such confinement was unlawful (e.g. not in the course of a valid arrest). The victims must be fully confined, and they are not required actively to search for an escape route. In this case, although there were no physical walls confining the couple, without clothing, they were confined by the choice of staying in the water or facing public nudity--there was no reasonable means for them to escape confinement from the water. Moreover, the facts state that the security guards intentionally took the couple's clothing, even though they knew the couple were in the lake. Therefore, although the couple did eventually discover a way out of their predicament, the security guards are guilty of false imprisonment.

QID: 9234 An employer refused to hire a disabled applicant. The applicant sued the emploeyr for violating the state's Disabled Employees Act and the Americans with Disabilities Act. The state law was analagous to the federal law, but had different filing and timeing requirements. The state trial court found in favor of the employe, but the state appellate court reversed, finding that the employer had violated federal law. The state supreme court affirmed the reversal. The employer filed a petition for a writ of cert. Should the U.S. Supreme Court grant the petition? (A) Yes, because Article III, Section 2 of the Constitution gives federal courts jurisdiction over federal questions. (B) Yes, if the state supreme court was unclear as to whether its decision was based on adequate and independent state grounds. (C) No, because under the abstention doctrine, the U.S. Supreme Court should allow the state court to resolve issues of state law. (D) No, because adequate and independent state grounds supported the state court judgment.

(A). Here, the state supreme court must have decided the case on federal grounds (in that it would have had to rule one way or the other on the federal ADA claim), and, furthermore, we are told that the state law was analogous to the federal law. The U.S. Supreme Court would have jurisdiction over this case based on the presence of a federal question, which is the most accurate reason why the U.S. Supreme Court should grant the petition. Here, the facts do not leave open the possibility that the case was decided on independent and adequate state grounds, because the state law is analogous to the federal law, with only procedural differences. As such, this is not the best explanation for why the U.S. Supreme Court should grant the petition.

QID: 10385 A store owner operates a retail store specializing in ethnic crafts and other tourist items. The store owner executed a writing contract in March with a supplier whereby the supplier would deliver 1,000 hand-carved fertility statues, a popular item with the seasonal tourist crowd. Delivery was set for May 1, and the contract price was $1000 for the lot. In early April, the supplier called the store owner to let him know that the wood carvers had unexpectedly gone on strike. The supplier wasn't sure that he would be able to deliver all of the 1000 pieces on May 1 as scheduled. Because there was still time for the store owner to find an alternative distributor, the store owner asked the supplier during that same phone call for assurances that the supplier could deliver all of the 1000 statues on schedule. If not, the store owner wanted to cancel the contract. The supplier said that he'd have to check. The next day, the supplier called the store owner and told him that he would do his best to have the 1000 statues on schedule, one way or another. Still insecure after the supplier's uncertain spoken assurance, the store owner called the supplier on April 12 and told him that he would be going elsewhere for his statues. He then ordered 1000 statues from a different distributor. On April 15, the store owner received a letter from the supplier asking for assurances that the store owner would accept delivery of the statues on May 1. The store owner called the supplier and reiterated that he was not going to be getting his statues from the supplier. On April 20, the supplier filed suit for damages for breach of contract. If the store owner retracts his repudiation on April 25, who is likely to prevail in the lawsuit? (A) The supplier will prevail, because the store owner anticipatory repudiated the contract. (B) The supplier will prevail, because the store owner breached the contract. (C) The store owner will prevail, because the supplier anticipatory repudiated the contract. (D) The store owner will prevail, because the store owner retracted his repudiation prior to the contract delivery date.

(A). In this case, the written contract was for the sale of goods and, therefore, the provisions of the UCC are applicable. The UCC requires that demands for adequate assurances be in writing. The store owner asked the supplier for assurances over the phone and therefore, did not satisfy the requirements under the UCC. As a result, although he did receive a response form the supplier, even if the supplier had failed to give any assurance, the store owner was not in a position to treat the supplier as though the supplier had anticipatory repudiated the contract. He had no grounds to cancel the contract with the supplier and, therefore, when he indicated to the supplier that he would not accept the supplier's statutes, it was the store owner who was threatening anticipatory repudiation. By asking for assurances in writing, the supplier satisfied the requirements of the UCC. Therefore, when the store owner confirmed that he was not going to honor the contract, the supplier was in a position to treat the situation as a breach of contract and sue for damages or specific performance. Once an aggrieved party has filed suit, it is too late for the repudiating party to retract the repudiation of the contract so the store owner's action on April 25 has no effect.

QID: 9225 Businesswoman's company provided maid and concierge service at time-shares and vacation rentals. Her employees stocked the properties with groceries prior to the guests' arrival, handled dinner reservations, and obtained tickets for shows, events, and tourist attractions. One day, Employee, in a hurry to get to the next job site, forgot to lock the door to Guest's rental house. When Guest arrived, she found the door unlocked and the movable appliances (TV, DVD player, stereo, and microwave) gone. She contacted the police. Property Owner later sued Busiensswoman for his losses. Who will prevail? (A) Property Owner because Busiensswoman's employee failed to use due care. (B) Property Owner, because he did not hire Busiensswoman's company to provide services to his renters (C) Businesswoman, because the contract was between Busiensswoman and Guest, so Property Owner was not a party to the contract. (D) Businesswoman, because the burglar's act was an independent superseding cause.

(A). It was clearly foreseeable that if the maid service company acted negligently, Property Owner might be harmed. Thus, because Property Owner was a foreseeable plaintiff, Businesswoman's company owed him a duty of care despite the fact that he was not a party to the contract between the company and the guest. Employee's act of leaving the door to the property unlocked increased the risk that someone would enter and steal the items inside. For this reason, Businesswoman is vicariously liable to Property Owner for the injury Employee caused.

QID: 9144 A state legislature enacted the Health and Safety in Marriage Testing and Education Act, which required all persons applying for a marriage license to undergo a screening for human papilloma virus (a sexually transmitted disease that is frequently asympotmatic and can lead to cervical cancer) and for hepatitis C (a communicable disease that can lead to liver failure). Under the Act, all applicants for a license are also required to attend a Saturday class to learn how to prevent the spread of these diseases from one partner to the other. The purpose of the act is to provide prospective spouses with information about two diseases that have severe health consequences and to slow the rates of the diseases in the state population. A resident of the state applied for a marriage license, but his application was denied because he refused to submit to testing for the two diseases, although he was willing to attend the class. If the resident challenges the act in federal court on constitutional grounds, who bears the burden of proof? (A) The state, to show that there is a compelling government interest in testing and educating prospective spouses about the two diseases. (B) The state, because the act substantiallya ffects the class of people seeking to marry. (C) The residence, because legislative enactments are presumed constitutional. (D) The resident, to show that the state has overstepped its Tenth Amendment police power to regulate for the health, safety, and welfare of its citizens.

(A). Laws affecting fundamental interests are subject to the strict scrutiny test, under which the state has the burden to persuade the court that it has a compelling interest in infringing on a constitutional right. The resident's challenge to the act is based on an allegation that infringes on the fundamental right to marry, so the state has the burden of showing a compelling need for the testing and education requirements of the act.

QID: 101189 A woman sued her doctor in federal court on a medical malpractice claim, alleging that her daughter's cerebral palsy was due to inappropriate care during delivery. At the jury trial, the jury was filing in to court after a recess when the defendant's niece, who was attending for the afternoon, saw that one of the jurors was a former classmate of hers. The defendant's niece waved to the juror and the juror waved back. After a conference in chambers, the judge decided to dismiss the juror, even though the juror said that he did not know the defendant's niece well and would not be prejudiced by the relationship. The court informed the jury that the juror had been dismissed but had not done anything wrong and did not specify further. The woman's lawyer moved for a mistrial, which was denied. The trial went on for three more days, and the jurors deliberated for two days. When the jury gave a verdict for the defendant, the woman's lawyer requested permission to interview the jurors to see whether they were affected by the dismissal of the juror. The court also denied that request. On appeal, what would be the hardest issue for the woman's lawyer to address? (A) That the court's error affected the woman's substantial rights. (B) That the woman's lawyer preserved the issue for review. (C) That the issue was on the record of the trial court's proceeding. (D) That the court's denial of juror interviews was clearly erroneous

(A). Not all types of error are review able on appeal. To be revised, an error must be preserved by being raised when it occurs, and it must be on the trial record. The court is likely to find that the woman met both of those requirements. However, the challenge here would be for her lawyer to show that this was more than "harmless error" and that it affected the woman's substantial rights. The facts given do not show that there was a likelihood of jury influence.

QID: 10359 A house painter decided to get a dog. Before she could adopt one, however, she needed to fence in her back yard. She had lived next door to the same neighbor, a seamstress, for 5 years. The house painter consulted with the seamstress about where the property line between their houses was located. Neither one of them was certain but since there was only fifteen feet between their houses, they just agreed to split it down the middle. The fence was built and the house painter got her dog. Seventeen years later, the house painter still lived on the property but the seamstress had recently sold her house and moved. The fence was in serious need of repair so the house painter decided to replace it altogether. She had the fence removed but when the fence company came to erect the new fence, the new neighbor stopped them and claimed that it would have to be moved two feet closer to the house painter's house because for all these years, the old fence had been encroaching on his property. He'd discovered the mistake with a survey. The house painter disagreed and furthermore, did not want to move the fence because she was growing award-winning roses right where the new neighbor wanted her to move the fence. If the neighbors go to court to settle the boundary dispute, who will prevail? (A) The house painter will prevail because the boundary line became fixed by the agreement between the house painter and the seamstress seventeen years earlier. (B) The house painter will prevail by adverse possession if the statutory period has been met. (C) The neighbor will prevail because he can prove the real property line with a survey. (D) The neighbor will prevail because the agreement between the house painter and the seamstress was not put in writing.

(A). Oral agreements to settle a boundary dispute are enforceable if the parties subsequently accept the line for a sufficient time. The seamstress and the house painter did not know where the line was located so they agreed on a boundary line and for seventeen years that agreement stood. The court will very likely find that enough time has passed since the agreement to make it enforceable in this case. When the new neighbor bought his house, he took it subject to that agreement. If the survey was performed before closing, showing that the fence appeared to be in the wrong place, he could have negotiated a different price with the seamstress but he may not force a change in the boundary line now. Because the boundary line in question was settled by agreement, the requirements for adverse possession are not met.

QID: 77286 Ten autoworkers believed that they were victims of racial and sexual discrimination because their employer had a decades-long general policy of only promoting white men into management positions. The autoworker group was comprised of three white females, two black mates, three Hispanic females, and two Asian males. The autoworkers had worked at the company from three to 25 years. Can the autoworkers join as plaintiffs to sue their employer? (A) Yes, because the autoworkers' claims arise out of the same transaction or occurrence. (B) Yes, because a plaintiff is the master of his or her claim. (C) No, because all of the autoworkers would have suffered discrimination for different reasons. (D) No, because the autoworkers were not all discriminated against at the same time because of the different amount of years they had worked at the company.

(A). Permissive joinder is used by plaintiffs who want to join together as multiple plaintiffs or want to join multiple defendants. Multiple persons may be joined together as plaintiffs or defendants in one action if: 1) their claim arises out of the same transaction or occurrence; and 2) any question of law or fact is common to all plaintiffs. Here, since the autoworkers are claiming a "decades-long general policy" of discrimination, a court would likely find that all their claims arise out of the same transaction or occurrence despite the fact that they all faced individual acts of discrimination.

QID: 9277 A defendant was charged with the crime of maintaining an illegal gambling house. During the defendant's trial, the state legislature amended the statute that criminalized maintaining a gambling house and changed the maximum punishment under the law from five years in prison to three years. A jury found the defendant guilty. During the defendant's sentencing, the judge stated that he had planned to give the defendant a five-year prison sentence, but because the law had changed during the trial, he would sentence the defendant to three years in prison. The state prosecutor appealed the sentence, asserting that to retroactively apply the legislature's change in sentencing provisions would violate the Ex Post Facto Clause of the Constitution. How should the appellate court rule? (A) In favor of the defendant, because a law that retroactively decreases a criminal penalty is not an ex post facto law. (B) In favor of the defendant, because the failure to grant a new trial due to the change in the law violated the Due Process Clause of the Fourteenth Amendment. (C) In favor of the state, because a law that retroactively changes a criminal penalty is an ex post facto law. (D) In favor of the state, because criminal laws may not be applied retroactively.

(A). The Constitution bars the federal and state governments from passing any ex post facto criminal laws. A statute is considered impermissibly to alter a criminal law retroactively if it (1) makes criminal an act that was not a crime when committed; (2) prescribes greater punishment for a crime after its commission; (3) decreases the amount of evidence required for conviction; or (4) extends the limitations period for a crime as to which a previously applicable limitations period has already expired. Inasmuch as the law in this question decreased the punishment for maintaining a gambling house, it is not an ex post facto law.

QID: 10388 Recently, there has been a trend for the makeup companies to market their wares to teenage girls while making the compacts, mascara wands, and the like look like miniature alcohol bottles. The U.S. Congress passed the "APT" -- Alcohol Promotion Tax, which is levied on makeup manufacturers who make their products look like alcohol bottles. Although labeled as a "tax," it seems clear that the APT was designed to impose a penalty on makeup manufacturers. A group of manufacturers challenges the APT's constitutionality. Is the APT constitutional? (A) Yes, because the U.S. Congress may use its taxing power as a regulating device rather than for revenue raising purposes. (B) Yes, because the modern judicial trend is to uphold any tax passed by the U.S. Congress as valid. (C) No, because the purpose of the tax is to regulate, not raise revenues. (D) No, because the tax is imposed on all manufacturers of makeup whose products look like alcohol bottles, not just those targeted at teenages.

(A). The U.S. Congress has the power to lay and collect taxes to pay the debts and provide for the common defense and general welfare. However, Congress sometimes uses its taxing power to achieve a regulatory effect. Generally, as long as Congress has the power to regulate the activity taxed, the tax can then be used as a regulating device rather than for revenue-raising purposes. Further, even when Congress does not have power to regulate the activity taxed, the tax will be upheld if the tax does, in fact, raise revenue (even where it has a substantial regulatory effect). Therefore, even though the effect (and perhaps even the purpose) of the APT is to regulate the promotion of alcohol and penalize manufacturers who promote alcohol, it will still be valid.

QID: 9135 A truck driver had just finished delivering a load of grapefruits and was exiting the freeway on his way home. A taxi collided with the truck at the freeway exit, which was marked as a four-way stop. The truck driver sued the taxi driver. A mechanic at a truck service center located at the freeway exit witnessed the accident but had apparently moved out of state and was unavailable to testify at trial. At trial, the truck driver called a jogger as a witness, who testified that she was near the scene of the accident but did not see it. She testified that she heard the mechanic yell that a taxi had run through the stop sign and had hit a truck. The taxi driver called as a witness a brake repairmen who worked with the mechanic. He testified that the mechanic told him the next day that the truck had run through the stop sign. The truck driver objected to this testimony. Should the brake repairman's testimony be admitted? (A) Yes, but only for impeachment purposes. (B) yes, as both substantive evidence and for impeachment purposes. (C) No, because it is hearsay not within any exception. (D) No, because the mechanic is not available to testify about the inconsistent statement attributed to him.

(A). The brake repairman can testify about the mechanic's statement only to impeach the jogger's testimony. The jogger's testimony as to what the mechanic said at the scene of the accident is admissible under the Federal Rules of Evidence hearsay exception for excited utterances. A declarant's hearsay statement may be attacked under FRE 806 through introduction of an inconsistent statement. Therefore, because the mechanic's first statement was admissible under a hearsay exception, the brake repairman could testify as to what the mechanic later said about the accident to impeach the jogger's testimony about the mechanic's earlier statement.

QID: 9111 A defendant is on trial for an assault on the night of March 23. The victim claims that he clearly saw the defendant's face by the light of the full moon. The prosecutor requests that the court take judicial notice of the fact that the moon was full on March 23. If the court does take judicial notice of the fact that the moon was full on March 23, which of the following is true? (A) The prosecutor has satisfied the requirement of producing evidence on the fact judicially noticed. (B) The judicial notice conclusively established the fact. (C) The court has erred, because this is an inappropriate matter for judicial notice. (D) The burden shifts to the defense to prove that the moon was not full on March 23.

(A). The court may take notice of any fact that is not subject to reasonable dispute. For a fact to be judicially noticed, the fact may be established by evidence. In some cases, a court or judge may take judicial notice of the fact without the evidence of such facts. In a criminal case, where a fact is judicially noted, the judge would instruct the jury that it may, but need not, accept a judicially noticed fact as having been proved beyond doubt. As such, this answer is correct. In a civil case, where a fact is judicially noted, the jury shall be instructed to accept such fact as being conclusively proved. However, in a criminal case such as here, the judge should instruct the jury that it may, but need not, accept a judicially noticed fact as having been proved beyond doubt.

QID: 28817 A man was arrested for possession with the intent to distribute a large amount of cocaine that was found in an abandoned vehicle. As he was being placed into the police car, he said, "Hey fellas, can't we work this one out? Sure the drugs are mine, but I bought them for personal use. I'm not a seller, I was gonna have a big party this weekend." At trial, the prosecution offers the man's statement. Is the defendant's statement admissible? (A) Yes, as a statement by an opposing party. (B) Yes, because the man had not requested a lawyer. (C) No, because the statement is privileged as part of plea negotiations. (D) No, because the officer failed to inform the man of his right to remain silent.

(A). The man is in custody. However, there is no questioning taking place; rather, this is a one-sided conversation. Miranda warnings are not required in this situation. Therefore, there are no procedural inaccuracies to suppress the satement. There are no valid privileges that man can assert, either. As such, his statement may be used against him and will be admissible as a statement by an opposing party. The man was in custody; however, the Fifth Amendment and the right to remain silent under Miranda also require that the police interrogate the defendant.

QID: 9119 A landowner sold her estate to a cousin and gave him a general warranty deed. Six years later, the cousin sold the property to a neighbor at a discount. The cousin gave the neighbor a special warranty deed that included no future covenants. The estate was very large, and fifteen years had passed before the neighbor noticed the house that a family of fisherman had erected on a pond forty years earlier. The neighbor sued to eject the fisherman, but the court ruled that the fisherman owned the property under the rules of adverse possession. The loss of the fishermen's property caused the neighbor a considerable decrease in the value of the estate. The statute of limitations for all lawsuits is six years. The adverse possession period is fifteen years. The neighbor comes to you for advice. How do you advise him? (A) The neighbor cannot collect damages from the cousin, but he can collect form the original landowner. (B) The neighbor cannot collect damages from either the cousin or the original landowner. (C) The neighbor can collect damages from the cousin, but he cannot collect from the original landowner. (D) The neighbor can collect damages from both the cousin and the original landowner.

(A). The statute of limitations on suits based on present covenants starts to run when title passes. In this case, the cousin did not make any future covenants on the property. Since the statute of limitations has clearly run, the neighbor is precluded form collecting damages from the cousin. Therefore, the neighbor's remedies in a suit against the cousin are limited to damages for breach of present covenants. However, the original landowner did make future covenants. The statute of limitations does not begin to run on future covenants until the grantee's rights have been disturbed by the grantee becomes aware of the encroachment. Therefore, the neighbor can collect damages form the original landowner for the breach of the covenant of quiet enjoyment.

QID: 9154 A wealthy businesswoman owned several acres of land. As a gift to her much younger sister, the businesswoman divided her property in half and gave her sister the portion of the land that faced south (Lot A), while the businesswoman retained the northern portion (Lot B). Lot A was bordered by a small private dirt road. Lot B faced a large public street. The businesswoman granted her sister an easement over Lot B so that she would easily access the paved road. The easement was written into the sister's deed and duly recorded with the appropriate authorities. After many years, the sister grew up, married, and built her home on Lot A. Her driveway existed onto the dirt road. However, she and her children used the easement to reach the public streets and school bus stops on Paved Avenue. When the busiensswoman died, she left Lot B to her sister for life, with the remainder to her own children. The sister, devasated at the loss of her older sister, died a year later. She left her property (Lot A) to her son. The businesswoman's children immediately moved into the house on Lot B and built a fence across the easement, making it clear that they did not want their cousin crossing their land. The sister's son filed suit against them demanding that the fence be removed and that he be allowed to cross Lot B using the easement. Will the sister's son prevail? (A) Yes, because he inherited the easement when he inherited Lot A from his mother. (B) Yes, because he has an easement by prescription. (C) No, because the easement was extinguished when the sister held both lots. (D) No, because he has access to public streets by way of the dirt road.

(A). This is a particularly difficult question. The sister's son inherited Lot A from the sister. The deed to Lot A contained an express easement over Lot B. As such, the sister's son inherited the easement with the property. Many students mention that for one year, the mother had a life estate in the servient estate--that is, she had a possessory interest in both estates. Therefore, is there no termination of the easement if the same person has a fee simple interest in one but only a life estate in the other? In the facts, the businesswoman gave her sister a deed and into that deed was written an easement across parcel B. That easement was to last for infinite duration. In order to extinguish the easement fully, the sister needed to have complete unity of ownership in both parcels A and B. However, she did not receive complete unity of ownership of both parcels. The sister received only a life estate interest in Lot B. As a result, the unity of ownership (in both parcels) lasted no longer than the lifetime of the sister, and so the easement was lost (through merger) only during this time. Therefore, when the sister died, the easement continued to exist and the sister's son can cross Lot B. As such, choice (A) was the best choice here. Generally speaking, merger is usually a means to extinguish an easement. But those situations involve the same and transfer of a fee interest in the servient tenement itself. This question took a different approach and only granted a life estate interest int he servient tenement.

QID: 9284 A coastal state is a popular tourist destination. Recently, on three separate occasions, tourists have been robbed at gunpoint at isolated highway rest areas in the state. In the most recent incident, the robber shot and killed one of the victims. On each occasion, a white sedan with a dent in the passenger side door was sighted in the vicinity. After the robbery that ended in murder, a trucker who saw the sedan speeding away gave the police a license plate number. the police tracked the license plate number and identified the driver of the sedan. The driver was also captured on an ATM security camera when he attempted to use an ATM card belonging to one of his victims. After obtaining an arrest warrant, police officers went to the driver's home, where he lived with his parents. When the police officers pulled up in front of the house, the white sedan was sitting in the driveway. As they approached the front door, the officers could see the driver through the front window in the living room watching television with his parents. When the driver answered the door, the police officers arrested and handcuffed him. Then, one of the officers proceeded to walk through the house, looking in all of the rooms and the garage. In the garage, the police officer found a box on top of the washing machine containing a gun, several wallets, and three or four cameras. At the driver's subsequent rial for robbery and felony murder, the prosecution sought to introduce the items found in the garage into evidence. The driver's attorney moved to suppress the evidence on Fourth Amendment grounds. Should the motion to suppress the evidence be granted? (A) Yes, because the police did not have a reasonable and articulable suspicion that accomplices were present in the house. (B) Yes, because police had probable cause that evidence of the robberies would be found in the house. (C) No, because the police officers were permitted to conduct a productive sweep of the house incident to the lawful arrest (D) No, because the driver's parents owned the house and the driver therefore had no reasonable expectation of privacy there.

(A). While police officers are permitted to conduct a search incident to a lawful arrest, the area that may be searched must be limited to the person arrested and the area within the person's immediate control (i.e. "wingspan") unless the officers have a reasonable and articulable suspicion that accomplices who might threaten the police are on the premises. Here, the police did not have a reasonable and articulable suspicion that dangerous accomplices were present in the house. Therefore, the search was unconstitutional.

QID: 9206 Lou died, leaving a $400,000 estate. Two years before his death, Lou made a will in which he left $50,000 to his niece, Georgia, and the remainder of his estate to his son, Chris. The will was on file with Lou's lawyer when he died. Georgia contests the will, claiming that Lou made a second will one month before his death. According to Georgia, Lou let her read the second will and told her that he was putting the original of the will in his safe deposit box for safekeeping. Lou did not give Georgia a copy of the second will. Despite a diligent search, Georgia and the lawyer have not been able to find either the safe deposit box or the second will. Georgia wants to testify that, in the second will, Lou devised one-half of his estate to Chris and the other half of his estate to her. Is Georgia's testimony admissible? (A) Yes, because Georgia can show that the second will has not been located despite a reasonably diligent search and not through any bad faith on her part. (B) Yes, because Georgia is not seeking to prove the contents of a writing. (C) No, because Georgia's testimony is hearsay not within any exception. (D) No, because admitting Georgia's testimony would violate the original document rule.

(A). With some exceptions, Federal Rule of Evidence 1002 ("Best Evidence Rule") requires a proponent to provide the original of a writing, recording, or photo when seeking to prove its contents. For the rule to apply, the proponent of the writing must be trying ro prove the contents of the writing, not merely that it exists, was delivered, was executed, or that a transaction was memorialized by a writing. The rule applies when a party is trying to establish the terms or working of a legally operative document, such as a will or contract. In such a case, the proponent must produce the original writing unless the proponent can establish that the original is unavailable. (Note, however, that a duplicate, as defined by FRE 1001, is generally admissible to the same extent as an original). Georgia is trying to prove the contents of a will. Ordinarily, she would be required to produce the original of the second will or a duplicate to prove its contents. However, the original of the second will has not been located. Under FRE 1004, secondary evidence of the contents of a writing is admissible if the originals are shown to be lost or destroyed, "unless the proponent lost or destroyed them in bad faith." Generally, the proponent will also have to show that he or she engaged in a reasonably diligent search. Thus, this answer is correct. Georgia's testimony is admissible as long as she can show that the original has not been located despite a reasonably diligent search and not through any bad faith on her part Hearsay is a statement made outside the court by a declarant that is offered to prove the truth of the matter stated. Georgia's testimony would not be hearsay because it is not about a statement made out of court. Georgia saw the second will herself when Lou showed it to her. Thus, she would be able to testify directly about what the second will said.

QID: 9163 A housewife is on trial for murder. In the jurisdiction in which she is being tried, murder has three elements: 1) an unlawful killing; 2) of a human being; 3) with malice aforethought. The jurisdiction's murder statute further provides that that self-defense is an affirmative defense. The housewife claims that she mistakenly thought that the person she is accused of killing was an animal, not a human being. She also claims that she acted in self-defense. At trial, the judge instructs the jury that the housewife must prove mistake-of-fact and self-defense by the preponderance of the evidence. Which, if any, of the judge's instructions were proper? (A) The mistake-of-fact instructions as proper. (B) The self-defense instruction was proper. (C) Neither instructions as proper. (D) Both instructions were proper.

(B). A defendant may be required to prove an affirmative defense by the preponderance of the evidence. Here, given the statute providing that self-defense is an affirmative defense, it was proper for the judge to instruct the jury that the housewife was required to prove self-defense by a preponderance of the evidence. However, the prosecution in a criminal case is constitutionally required to prove every element of a crime beyond a reasonable doubt. Given that the jurisdiction's murder statute specifically provides that one of the elements of the crime of murder is that the defendant intended to kill a human being, the prosecution is required to prove that the housewife intended to kill a human being. Thus, the mistake-of-fact instruction unconstitutionally shifted the burden of proof to the defendant. For this reason, it was not proper for him to instruct the jury that the housewife was required to prove mistake-of-fact by the preponderance of the evidence.

QID: 9272 A boyfriend and girlfriend owned their house as joint tenants. After several years, the boyfriend decided to leave the girlfriend. He sold his interest in the house to his sister. The day after the transfer to his sister, the boyfriend died. What is the state of the title to the house? (A) The girlfriend owns the house as the surviving joint tenant. (B) The girlfriend and the boyfriend's sister own the house as tenants in common. (C) The girlfriend and the boyfriend's sister own the house as joint tenants. (D) The boyfriend's sister owns the house in fee simple.

(B). A joint tenancy is a form of co-ownership in which each tenant owns an undivided interest in the whole state. The main characteristic of this type of tenancy is the right of survivorship--when one joint tenant dies, ownership of the entire property passes to the surviving tenant with nothing passing to the heirs of the decedent. However, if one party transfers his share inter vivos, the joint tenancy is severed. When the boyfriend sold his share to his sister, the joint tenancy between the boyfriend and girlfriend was destroyed, and the girlfriend and the boyfriend's sister then became tenants in common. As such, this answer is correct.

QID: 10394 A sister and brother bought a farm together. It was large and surrounded by trees, and they were blissfully unaware of activities on the adjacent parcels of land. Unbeknownst to them, the adjacent parcel included an underground graphite mine. Although the mining took place only directly below the adjacent parcel, years of digging had weakened the surrounding areas, including a cave under the farm. Several years after the brother and sister moved in, an uncultivated portion of the farm they used for pasture sank. The mine owner was unresponsive to their requests for compensation, and the brother and sister filed suit. What will be the result of the suit? (A) The brother ans sister will prevail, if the mine owner was negligent. (B) The brother and sister will prevail, regardless of whether the mine owner was negligent. (C) The mine owner will prevail, as long as all mining regulations were followed. (D) The mine owner will prevail, because there were no improvements on the damaged land.

(B). A landowner who engages in activities that cause the subsidence of unimproved land on an adjacent parcel is strictly liable for resulting damage. Thus, here, the brother and sister will prevail against the landowner whose mining damaged their property. If the damaged property is unimproved, the plaintiff need not show negligence. If the damaged property IS improved, the plaintiff will need to show the defendant was negligent.

QID: 9238 A hardware company manufactured one of the most powerful electric hand drills on the market. The hand drill was frequently used by professional carpetners. When set at its highest speed, the drill could be difficult to control with just one heand, so the hardware company put a sticker on each drill, which read: "This drill is a high-speed power tool. Always use great care when working with this or any other power tool." A woodworker bought the company's drill for a construction project at his home. The woodworder set the drill to its highest spped, but used only one hand to operate it, as he was accustomed to doing with his smaller, less powerful hand drill. The drill fell from the woodworker's hand and landed on his foot, breaking two of his toes. Which of the following is the most accurate? (A) The woodworker should prevail in a products liability suit against the hardware company, because the drill had a manufacturing defect. (B) The woodworker should prevail in a products liability suit against the hardware company, because the company should have put a clearer warning on the drill. (C) The woodworker should not prevail in a products liability suit against the hardware company, because he did not exercise adequate care in using the drill. (D) The woodworker should not prevail in a products liability suit against the hardware company, because the hardware company placed a warning on the drill.

(B). A product may be dangerously defective because it is accompanied by an inadequate warning--that is, it fails to communicate its message to the user, fails to sufficiently describe the danger, fails to mention all dangers, or is inconsistent with the instructions for use of the product. In this case, the hardware company should have specifically warned that it was necessary to use two hands to operate the drill on its highest speed, rather than just including a general statement about exercising care when using the drill. Because the warning on the drill failed to sufficiently describe the danger, the woodworker should prevail in a products liability suit against the hardware commpany.

QID: 9281 Cara, a con artist, contracted to sell Andrew a large piece of property. However, Cara did not hold title to the property. Under the terms of the purchase and sale agreement, Andrew was to pay Cara $100,000 for the property. On the closing date, Andrew tendered a cashier's check for $100,000. Cara properly executed a quitclaim deed to the property and gave it to Andrew. Several months later, Andrew's neighbor, Lora, decided that the property would be the perfect location for a store she planned to open. Ora offered to buy the property from Andrew for $125,000. Andrew agreed and executed a warranty deed, which he gave to Lora. Lora paid Andrew $125,000 a week later by certified check. Lora then checked the records and found that Andrew did not hold title to the property. She immediately complaint to him. Andrew then checked the records himself and learned that Lora was correct. Andrew immediately called Cara and threatened to sue if Cara did not produce good title to the property. Cara then bought the property for $90,000 and promptly executed a deed to Andrew, who called Lora and reported to her what has happened. Andrew further assured Lora that because he now had a valid deed to the property, Lora now held good title to the property. If Lora nevertheless sues Andrew, who will prevail? (A) Lora, because Andrew did not hold marketable title when the deed was executed. (B) Lora, because of Andrew's breach of warranty. (C) Andrew, under the doctrine of estoppel by deed. (D) Andrew, because Lora's suit should properly be against Cara, who was the one guilty of fraud and unjustly enriched.

(B). A warranty deed includes a covenant that the vendor holds the interest conveyed (i.e., the covenant of seisin). If the vendor does not actually hold the interest conveyed, the purchaser can sue for any damages resulting form that breach of covenant. In this case, Andrew received the property by quitclaim deed (receiving whatever interest Cara had) and conveyed the property to Lora by warranty deed (warranting that there are no defects in the chain of title). As such, Lora can sue for any damages she has suffered as a result of Andrew's breach of the covenant of seisin. Because Andrew has, in fact now acquired title by virtue of the doctrine of estoppel by deed and can now convey good title to her, it is unlikely that Lora will recover the entire purchase price. However, she will likely be entitled to recover at least any additional costs required to confirm her title, such as the cost of recording a second deed or legal fees. In addition, if Lora has incurred other damages (e.g, the loss of the retail business she had envisioned) because of her temporarily not having good title, she will likely be entitled to recover those damages.

QID: 21785 A man is on trial for homicide. The man is accused of killing his boss, and a prosecution witness is prepared to testify that he heard the defendant at a diner bragging about how he killed his boss. After his preliminary hearing testimony, the prosecution's witness disappeared and the defendant's gang is implicated in getting rid of him at the defendant's request. The prosecution seeks to introduce the witness's preliminary hearing testimony concerning the defendant's statement. Over the defendant's objection, the court allows the testimony and the defendant is convicted. Was the trial court's ruling proper? (A) Yes, because the witness's statement constitutes former testimony. (B) Yes, because the defendant caused the declaration's unavailability (C) No, because the statement is hearsay not within any recognized exception. (D) No, because the testimony violates the Confrontation Clause.

(B). As this answer correctly points out, the Confrontation Clause is not violated when the defendant is responsible for the witness's unavailability. Therefore, the court's ruling was proper. While there is a possible hearsay exception for the witness's prior testimony, this answer fails to address the possibility that the Confrontation Clause has been violated. As such, this is not the best answer choice.

QID: 9282 A homeowner owns a house that he rents to a tenant on a month-to-month basis. In exchange for a reduction in the monthly rent, the tenant has agreed to perform any necessary yard work, including mowing the lawn and raking and disposing of leaves from several large sycamore trees on the property. Over the period of months that the tenant has been renting the property, the homeowner has become increasingly frustrated at the tenant's neglect of the yard work. After several unsuccessful attempts to contact the tenant by telephone, the homeowner decides to speak with him personally. The homeowner drives to the house to talk with the tenant, but the tenant is not there. The homeowner notices that the accumulated leaves in the backyard are beginning to kill portions of the lawn, and he decides to rake the leaves himself while he waits for the tenant to return. Because the day is quite windy, the homeowner realizes that it is not the optimal time to be raking leaves, but he does not have the patience to wait any longer. He rakes the leaves into piles and then lights the piles on fire in order to dispose of them quickly. The homeowner notices that the burning piles of leaves are very close to one corner of the house, but he is too angry with the tenant to take any precautions. While the homeowner is still in the process of raking and burning the leaves, the tenant returns home. Leaving the burning leaves unattended, the homeowner storms into the house to confront the tenant. While the homeowner is in the house, the wind blows burning leaves into the overgrown grass near the side of the house, setting the grass and the house on fire. The fire seriously damages on the corner of the house before it is extinguished by the fire department. The homeowner is charged with arson, for which the jurisdiction uses the common law definition of the "the malicious burning of the dwelling house of another." At trial, is the homeowner likely to be found guilty or not guilty? (A) Guilty because the homeowner entered upon the proeprty without permission while it was in the tenant's possession. (B) Guilty because the homeowner acted with reckless disregard for the obvious risk that the house would burn. (C) Not guilty, because the homeowner did not intend for the house to catch fire. (D) Not guilty, because the homeowner owns the house.

(B). Common-law arson is the malicious burning of another's dwelling. For this purpose, malice includes the reckless disregard of a high risk of harm. Here, the homeowner exhibited a reckless disregard of the clear risk that the burning piles of leaves would ignite the house occupied by the tenant. As such, he is guilty of arson. He need not have intended for the house to burn. It is true that the homeowner owned the house that was burned. However, it was the tenant who was in possession and had the legal right to occupy the property. As such, the homeowner may be guilty of arson despite the fact that he owned the property.

QID: 9115 After an anhydrous ammonia leak on a city railway line led to the death of three dozen people, the mayor had little trouble convincing the city council to ban the transportation of poisonous gases on that railroad line. Several other communities within the state followed suit, until carriers transporting gases could no longer traverse the state legally. The state is small and not centrally located, so other railway lines in neighboring states allowed the carriers to reach their intended destinations without an unreasonable burden. Although the inconvenience was minor, a group of carriers challenged the city ban in federal court. When the case reached the U.S. Supreme Court, the court applied the rational basis test to find that the city bans on transportation of lethal gases did not impose an undue burden. If Congress were thereafter to enact legislation prohibiting states from banning the transportation of lethal gases by rail or other channels of commerce, would the federal legislation be valid? (A) Yes, because the state law would have to give way to the federal law under the Supremacy Clause. (B) Yes, because the federal law, enacted under Congress's Commerce Clause power, intends to occupy the field. (C) No, because the highest court in the land has authorized the states to ban the transportation of poisonous gases within their borders. (D) No, because the equal protection rights of state carriers are violated by the federal prohibition against banning the transportation of poisonous gases within the state's borders.

(B). Congress has plenary power to regulate interstate commerce pursuant to the Commerce Clause, so its prohibition of state laws banning the transportation of poisonous gases is within that power and preempts any state law in direct conflict with it. The commerce Clause authorizes Congress to regulate the channels of interstate commerce, including railways. The Supremacy Clause provides that the Constitution and federal law are the supreme law of the land. The federal prohibition of state bans on transporting poisonous gasses by rail. Therefore, the city bans, which are in direct conflict with it, are void. As such, this is the best answer. The Supremacy Clause provides that the Constitution and federal law are the supreme law of the land. While it is true that a state law in direct conflict with a valid federal law is void pursuant to the Supremacy Clause, this answer does not state the source of power under which the federal law will be upheld. The Commerce Clause is the source of the congressional power to enact economic legislation that supersedes state law in the area of interstate commerce. Congress's power under the clause is plenary, and any federal legislation validly enacted under that power preempts conflicting state law.

QID: 10367 A native American reservation is located int he middle of a state. The reservation is large, and Native Americans actually occupy only a small portion of the reservation, which is rich in natural resources. Moreover, a large river flows through the reservation and many merchants use the river to ship goods between states. A foreign country has offered to purchase a portion of the reservation from the United States--Specifically, the portion not occupied by the Native Americans. However, the river flows through this portion of the reservation. By selling the land, Congress will be able to eliminate a substantial portion of the national deficit. Some of the surrounding states are upset over the possible sale of the land, and seek to block the sale. (A) Yes, because under a balancing test, Congress's interest in selling the land outweighs the states' interest in keeping the land. (B) Yes, because Congress has the power to dispose of property belonging to the United States. (C) No, because there is a compelling state interest to not sell the reservation. (D) No, because Congress does not have the power to sell a portion of a state.

(B). Congress has the power to dispose of any property or territory which belongs to the Federal Government (United States), including Native American reservations. Therefore, Congress does indeed have the power to sell the Native American reservation to a foreign country. Note that no balancing test is required, nor will a state's compelling interest prevent the sale. The power lies solely with Congress, as set forth in the Property Clause. Native American reservation is not a portion of the state. Even though it is located within the boundaries of the state, the reservatio itself is property belonging to the federal government.

QID: 10404 A motorcycle mechanic has more work than she can finish herself in a reasonable time. As she has done in the past, she subcontracts with her boyfriend, an accomplished mechanic in his own right, to finish work on two racing motorcycles owned by the same racer. The work is complicated and requires the boyfriend to buy many specialty parts. The purchase of the parts is anticipated and included in the contract price. The parts turn out to be a littler more expensive than anticipated, but the boyfriend sees no reason to bring it up to this girlfriend, figuring they'll work it out on another job. During the six weeks that the boyfriend works on the two motorcycles, the motorcycle mechanic pays him two installment payments towards the total contract price. When he is done, the racer/motorcycle owner is so pleased that he tips the boyfriend $50. Unfortunately, the motorcycle mechanic breaks up with the boyfriend during the six weeks, and when the boyfriend seeks his final payment under the completed contract, she refuses to pay him. If the boyfriend seeks legal damages in court, which of the following correctly describes his likely recovery? (A) The boyfriend will be limited to expectation damages, which in this case are the complete contract price plus the extra costs of the special parts, less the two installment payments and the $50 tip form the racer. (B) The boyfriend will be limited to expectation damages, which in this case are the complete contract price less the two installment payments. (C) The boyfriend will be limited to reliance damages, which in this case are the cost of the parts purchased and the market value of his services for the six weeks, less the two installment payments and the $50 tip from the racer. (D) The boyfriend may elect to recover restitutionary damages based on the market value of his services and the expense of the parts he bought, less the two installment payments.

(B). The boyfriend may recover expectation damages, which are designed to put him in the position he would have been in had the motorcycle mechanic fulfilled her part of the contract. In this case, the contract price is the measure of the expectation damages. The motorcycle mechanic has already paid two installments on the contract price, so those amounts will be deducted from the recovery. The boyfriend will not be able to claim any additional money to cover the higher-than-expected prices of the parts he purchased. The court will presume that the total contract price accounted for fluctuations in the price of anticipated parts and, as it turned out, the increase in price was modest. The tip paid by the owner of the motorcycles has no impact on the contract between motorcycle mechanic and the boyfriend so that $50 will not be deducted from his recovery. The boyfriend will not be able to elect restitutionary damages instead of expectation damages because his performance under the contract was complete. Restitutionary damages are limited to situations where the aggrieved party has only partially completed his obligation under the contract.

QID: 77270 A man, a citizen of Maine, sued a video game company in Maine federal court after the man suffered seizures while playing one of the company's games, Northeast Big Buck Hunter, a videogame that was specifically marketed and sold in the states of Maine, Massachusetts, New Hampshire, and Vermont. The man claimed over $1 million in damages. The game was a huge hit and sold millions of copies in those four states, with over 1 million sold, many of which were in Maine. The video game company was incorporated in California and had all of its offices in California. However, because of its failure to repeat the success of Northeast Big Buck Hunter, the company has been experiencing financial difficulties. Therefore, the company argued it could not afford to defend itself in Maine. Does the Main federal court have personal jurisdiction over the company? (A) Yes, because the company sold over 1 million games, many of which were in Maine. (B) Yes, because the company marketed and sold the games in Maine. (C) Yes, because the man was injured by the game in Maine. (D) No, because forcing the company to defend in Maine would offend traditional notions of fair play and substantial justice.

(B). The company sold and marketed the game in Maine, which created sufficient minimum contacts with the forum state. Based on the contacts found between the forum state and the defendant, the court must find that exercising personal jurisdiction over the defendant would not offend traditional notions of fair play and substantial justice. This is a balancing test used by the courts, weighing the following factors: 1) the plaintiff's interest in trying the case in the forum state and the availability of other forums; 2) the defendant's burden in trying the case in the forum state as opposed to the defendant's state of residence; and 3) the forum state's interest in the case. Here, just because the company is having financial difficulties will not be a sufficient reason for a court to not subject a defendant to personal jurisdiction. A court will likely find the defendant's burden in defending in Maine (when its offices are in California) is not so high as to offend traditional notions of fair play and substantial justice.

QID: 9263 A manufacturer of lamps and other household products has been buying materials for glass production from a supplier since she started her business. When the supplier told her that it was going to have to substantially raise its prices for the materials she used to produce the glass, the manufacturer searched for another supplier because she had to keep her prices low to compete. She eventually found another supplier and asked the supplier how much her glassmaking materials would cost, indicating that in the prior fiscal year, she had used 5000 pounds of material to make her lamps. A few days later, the lamp manufacturer received a letter from the new supplier stating that the materials could be supplied to the manufacturer at a rate of 85 cents per pound. Furthermore, the letter assured the manufacturer that the supplier could provide all the glassmaking material she needed. The manufacturer wrote back to the new supplier accepting the offer to supply her with glassmaking materials if the supplier would give her a discount of 5% if she paid cash within 15 days of the supplier's billing date. What is the best characterization of the manufacturer's letter regarding the 5% discount? (A) It is making a counteroffer, but not rejecting the new supplier's offer. (B) It is rejecting the new supplier's offer and making a counteroffer. (C) It is accepting the offer and proposing an additional term because both the manufacturer and new supplier are merchants. (D) It is accepting the offer and proposing an additional term, regardless of whether the manufacturer and the new supplier are both merchants.

(B). The manufacturer's letter about the 5% discount would be considered a rejection of the new supplier's offer and a counteroffer made by the manufacturer. Under the traditional rule, an offeree could only accept an offer by unconditionally agreeing with the exact terms of the offer (the mirror image rule). However, the Uniform Commercial Code does not follow the mirror image rule. UCC Section 2-207 states that a definite and seasonable expression of acceptance or a written confirmation sent within a reasonable time operates as an acceptance, even though it states terms additional to or different from the terms of the offer, unless the acceptance is expressly made conditional on the new supplier's acceptance of a 5% discount and a 15-day payment term, it will not be considered to be an effective acceptance under either the UCC or the common law.

QID: 10393 Cook was married to Baker. They owned a busy cafe that served lunch and dinner. For several years, they stole credit card numbers for sale on the black market. They did this by having their employees swipe cards through a machine that, unbeknownst to the employees, captured numbers and other information that Cook and Baker could sell. Cook handled all of the details. One day Cook asked his wife, Baker if they should consider relocating the cafe to a new city to find new customers to scam. Baker was not in favor of the idea and they did not discuss it further. A year or so later, they divorced after Cook caught Baker in the amorous clutches of Dishwasher. After the divorce, Cook moved the cafe to a different city in the same state. He set up the scam again and this time was caught. The prosecutors found out about the scam at the original cafe and arrested Baker as well. Baker claimed that she'd had no knowledge about the scam and that it was all Cook's doing. At Baker's trial, can Cook take the stand to testify about his former wife's involvement with and knowledge about the scam? (A) Cook can testify about Baker's actions but he cannot testify about confidential communications they had while they were married. (B) Cook can testify about the Baker's actions and about any confidential communications about the crimes that they had while they were married. (C) Cook cannot testify about Baker's actions but he can testify about confidential communications about he crimes that they had while they were married. (D) Cook cannot testify for the prosecution at Baker's trial.

(B). The spousal privilege, which prevents a spouse from being compelled to testify against their defendant spouse in a criminal trial, is lost once the couple is divorced. Therefore, Cook may testify about Baker's criminal actions. The marital communication privilege survives divorce. In any civil or criminal case, either spouse, whether or not a party, has a privilege to refuse to disclose, and to prevent another from disclosing, a confidential communication made between the spouses while they were husband and wife. But there are exceptions to the marital communication privilege. When the spouses were joint participants in a crime, the marital communication privilege does not apply with respect to communications about the crime. Therefore, Cook may testify about the conversations he had with Baker regarding their joint crimes.

QID: 10406 A birdwatcher enjoyed wading around the shore of the lake abutting the rear of his rural property. The lake was small and too shallow for boating but it supported a large variety of small fish and plant life that attracted many water birds. The birdwatcher's neighbor also enjoyed wading in the lake, but his primary purpose in using the lake was fishing. Eventually, the neighbor had a long dock built out into the lake from his backyard so that he could fish from the dock. He also portioned off a small part of the lack connected to his dock and began experimenting with fish farming. Several other people owned houses on this particular lake that was natural and isolated. After the neighbor built the dock, several people, including the birdwatcher, who had lived on the lack much longer than the neighbor, complained about the dock's intrusion into the lake. There are no applicable statutes or deed restrictions preventing a dock. If the birdwatcher files an action to force the neighbor to remove the dock, claiming a violation of riparian rights, what is the likely outcome? (A) The neighbor will prevail because his dock is primarily for domestic use. (B) The neighbor will prevail because he owns the lake bottom out to the middle of the lake. (C) The birdwatcher will prevail under the prior appropriation doctrine. (D) The birdwatcher will prevail because the domestic use of riparian rights is superior to agricultural use.

(B). Title to lands under a non-navigable body of water, such as this small isolated lake, belongs to the abutting landowner and stretches to the center of the body of water. Provided that there are no state laws or deed restrictions forbidding the building of a dock, the neighbor will prevail. Riparian rights do not apply to water in non-navigable bodies of water such as this one.

QID: 10363 Steve and Justin were doing maintenance to Justin's large sailboat. The boat was docked in a canal behind the home that Justin shared with his wife, two children, and his ailing mother. Steve had long claimed that the boat resembled a pirate ship, and more than once the two friends had pretended to be pirates. On this day, the friends took up a fake fight with their mops. They noticed that Justin's wife had come out into the yard, but they continued their mock battle which only ended when Steve pretended to whack Justin over the head and Justin "fell" off the boat into the water. Justin's wife, playing along, screamed. Justin's ailing mother, Rita, who was now standing unseen on the back porch, believed the injury to be real and suffered a heart attack, resulting in her having to be hospitalized. Can Rita recover from Steve for intentional or negligent infliction of emotional distress? (A) Yes, but only for intentional infliction of emotional distress. (B) Yes, but only for negligent infliction of emotional distress. (C) No, because she was not in the zone of danger. (D) No, because Steve was unaware of her presence.

(B). To recover for a direct claim of negligent infliction of emotional distress, a plaintiff must prove that she: (1) was in the zone of danger; and (2) suffered emotional distress and some accompanying physical manifestation. While Rita did suffer physical harm (she had a heart attack as a result of her emotional distress), she was not in the zone of danger. As such, she would not be able to recover in a direct claim for negligent infliction of emotional distress. However, she could recover as a bystander. To recover in a bystander action for negligent infliction of emotional distress, the plaintiff must prove that she: (A) was present at the scene and witnessed the event; (b) is a close relative of the third party/victim; and (c) suffered severe emotional distress. Note that the plaintiff is not required to be in the zone of danger, nor must the defendant be aware of the plaintiff's presence, in order to recover under this theory. Because Rita was present at the scene, witnessed it, is a close relative of the third party/victim, and suffered severe emotional distress, she will be able to recover in a bystander action for negligent infliction of emotional distress.

QID: 9246 When a customer went to a drugstore to fill a prescription, she slipped and fell on pills that had scattered on the floor. The customer sued the drugstore, alleging negligence. At trial, the customer's attorney called a witness who had seen the customer fall. The attorney asked, "What do you believe caused the customer to fall?" The drugstore's attorney objected on the ground that the question called for an opinion on an ultimate issue. The trial court sustained the objection. The customer's attorney did not take exception to this ruling or make an offer of proof. The witness was then excused from the witness stand without being cross-examined. The jury rendered a verdict for the drugstore. The customer appealed, alleging that the trial court erred in its ruling on the drugstore's objection. If the desired testimony was essential to the customer's case, how should the appellate court rule? (A) The trial court's ruling should be reversed because the witness was testifying as to a legal conclusion. (B) The trial court's ruling should be reversed because lay witness testimony can address ultimate issues. (C) The trial court's ruling should be affirmed because the customer's attorney did not take an exception to the trial court's ruling. (D) The trial court's ruling should be affirmed because the customer's attorney did not make an offer of proof.

(B). Ultimate issues, such as a defendant's culpability are to be resolved by the finder of fact, and not by either lay or expert witnesses. However, a witness may testify in the form of an opinion or inference as to an ultimate issue. The only exception to this is in a criminal case, where an expert witness testifying as to a mental state or condition may not state an opinion or inference as to whether a defendant possessed a mental state or condition constituting an element of or defense to a crime. The witness in this trial, therefore, should have been permitted to testify. Trial counsel is not required to take exception to a ruling of the court to preserve the issue of the ruling's correctness on appeal. The party raising an objection should make sure that her objection is preserved in case of appeal, but there need not be any formal expression of disagreement with the ruling being placed upon the record.

QID: 28821 A mother lived in a rural area. She had a four-year-old son who was extremely bright for his age. He would often leave the hosue and climb into the car, pretending to drive. One afternoon, when the other was unloading groceries with the car running in the garage, the child climbed behind the wheel. As he was pretending to drive, he accidentally hit the gear shift, puttting the car into neutral. The car rolled down the driveway, across a neighbor's lawn, and finally collided with a passing car. Neither the child not the other driver was injured, but there was damage to both the other driver's car and a portion of the neighbor's hedge. Both the neighbor and the other driver sue the mother for damages. Who is entitled to recover? (A) Both the neighbor and the other driver, because the child was involved in an adult activity, and each plaintiff suffered property damage. (B) Both the neighbor and the other drive,r because the mother was negligent, and each plaintiff suffered property damage. (C) The other driver only, because the child did not intend to enter the neighbor's property. (D) Neither because the child was too young to appreciate the nature and quality of his actions.

(B). The mother left the car running in the driveway, and was likely aware that she had a child who liked to climb into the car and pretend to drive. The mother's negligence in supervising her child will make her responsible for the child's actions, which caused the car to enter the neighbor's lawn and to hit another vehicle. The facts indicate that both plaintiffs suffered damages. Therefore, the mother will be held responsible for the damages caused to the neighbor's hedge and the other driver's car. Note that the question indicates that the plaintiffs sue the mother, not the child. Therefore, the fact that the child may or may not have been engaged in an adult activity is not the central concern of the question. Although a child behind the wheel of a car can sometimes subject the child to a higher duty, when involved in an adult activity, the facts do not indicate the child was actually driving. In any event, because the defendant is the mother, the correct answer to the question is the one that targets the issue of the mother's own negligence, not the child's.

QID: 9201 On the last day of its legislative session for 2009, a state legislature enacted the following statute, to go into effect on January 1, 2010: "Recycling creates many jobs in our great state, but many state citizens could benefit form increased demand for recycled products. Therefore, all offices in the state must use only 100% recycled paper in their copiers and printers." The governor signed the bill into law. On November 1, 2009, the state Chamber of Commerce filed a complaint in state court challenging the constitutionality of the new law. The state filed a motion to dismiss. Which argument would best support the state's motion? (A) The statute is constitutional, because the state's police power includes the authority to enact legislation for the health, welfare, and safety of its citizens. (B) The Chamber of Commerce does not have third-party standing. (C) The matter is not ripe for adjudication. (D) The complaint asks for an advisory opinion, which state courts are not permitted to render.

(C). A "case" or "controversy" is a real and substantial legal dispute that can be resolved by judicial decree. A controversy must be ripe for decision, so that courts do not waste time deciding constitutional issues that could turn out not to need deciding. Thus, a person asking a court to hold a statute unconstitutional must be able to show not only that the statute in question is invalid, but also that he has been, or is about to be, injured because of its enforcement. Here the statute was to go into effect on January 1, 2010, and the complaint was filed on November 1, 2009, so the issue was not yet ripe. This is the state's best argument for dismissal. An association has standing to assert its members' claims, even if the association itself has not suffered any injury if: (1) the members would otherwise have standing to sue in their own right; (2) the interest asserted is germane to the association's purpose; and (3) neither the claim asserted nor the relief requested would require participation by the individual members in the lawsuit. The Chamber of Commerce could arguably qualify for third-party standing under this test, so this is not the state's best argument for dismissal.

QID: 9112 A small shoe company was looking to expand its business and become a prominent name in fashion. To this end, the company decided to see if it could secure cutting-edge designs for new shoes. In September, the company published an announcement in several national fashion specialty publications stating that it would pay $30,000 to anyone who, before the end of the year, would produce "exciting new shoe designs in hemp and other fabrics." An aspiring designer saw the advertisement and decided to pursue the project. The day before she began work on the designs, the designer texted the shoe company to say that she was accepting its offer, but the shoe company never received her text. Two months later, the shoe company could no longer afford to continue as an entity and was acquired by a large corporation. In mid-December, the designer perfected her designs and demanded that the shoe company pay her the $30,000. The shoe company refused. The designer later entered into an agreement with a footwear manufacturer to sell her designs for $40,000. How did the designer's contract with the footwear manufacturer affect the shoe company's duty to perform? (A) The shoe company was released from any obligation to pay the designer the $30,000 when she sold the designs to the footwear manufacturer. (B) The shoe company's promise could be enforced only in equity, but it would be inequitable to pay the designer the $30,000 because she had received a larger sum from the footwear manufacturer. (C) It had no effect, because the shoe company was obligated to pay the designer when she developed the shoe designs as specified in the offer. (D) The shoe company had only promised to make a gift of money, so the promise could not be enforced in any manner.

(C). A unilateral contract is one that may be accepted by a party's performance. Here, the shoe company's advertisement was an offer for a unilateral contract, which the designer accepted by submitting shoe designs as specified in the advertisement. At that time, the shoe company's obligation to perform became absolute. The footwear manufacturer's later payment of $40,000 for the designs, therefore had no effect on the shoe company's duty to perform. The call of the question does not concern the amount of damages. It only asks what effect the subsequent payment has on the shoe company's duty to perform.

QID: 21782 A defendant is arrested and charged with a crime. At a pretrial motion to suppress, the defendant asserts that his arrest was unlawful because the police did not have a warrant. Under which scenario would the defendant's arrest NOT be valid? (A) The defendant was arrested for felony robbery. (B) The defendant was arrested for misdemeanor drug possession after purchasing narcotics from an undercover officer. (C) The defendant was arrested for misdemeanor drug possession after a concerned citizen called 911 and described the defendant and the transaction. (D) The defendant was arrested for public drunkenness, a misdemeanor that is not punishable by incarceration after officer sees him staggering in a street with a bottle

(C). A warrant is generally not required for an arrest. Warrantless arrests are permitted for felonies, whether committed in an officer's presence or not, as well as misdemeanors committed in an officer's presence. However, this scenario, which describes an arrest for a misdemeanor offense committed outside the officer's presence would not be proper.

QID: 77300 A woman sued a shopping mall for $100 in repairs after she fell over a new holiday display and broker her glasses. The display involved a tiny train circling the mall's sidewalks. The court ruled int he woman's favor and found the mall's use of the train was negligent. A man then came forward and said he had also fallen over the train. He sued the mall for $10 million. He asked the court to find the mall negligent in his case since the earlier case had already decided that the mall was negligent. Should the court do so? (A) Yes, because a court had already decided the issue of negligence. (B) Yes, because the mall was involved in the first lawsuit. (C) No, because the mall did not have incentive to litigate the issue in the first matter. (D) No, because the man was not involved in the first lawsuit.

(C). Collateral estoppel, or issue preclusion, prevents re litigation of issues that were fully and fairly litigated, and were necessarily decided in a proceeding that reached a final judgment on the merits. Issue preclusion may be used offensively by one who was not a party to the first action against one who was a party in the earlier suit. Courts are reluctant to permit offensive use of issue preclusion and will base their determination on the following: (a) whether the plaintiff in the second suit could have easily joined int he first action; (b) whether there are procedural opportunities available to the defendant in the second suit that were unavailable in the earlier action; and (c) whether the defendant had incentive to litigate the issue in the first action. Here, the most important issue is that the mall had very little incentive to defend itself against the first claim, since it was only for $100 repair bill. Allowing offensive use here would be unfair.

QID: 9287 A congressional study of the sports programs in public schools throughout the country verified anecdotal evidence that non-white students were discouraged form participating in certain sports, such as golf, tennis, and field hockey while being encouraged to develop skills in basketball, track, and soccer. Congress creates the commission for Non-biased participation in Sports to promote the active participation of all school-age children in every sport offered in the public schools. The commission is empowered to oversee the administration of sports programs of school districts and to order compliance with federal regulations promulgated by the commission. Headed by an executive director appointed by the president, the commission renders decisions on school programs brought before the commission for violation of federal regulations. Appeal is to the vice president, subject to judicial review by the federal courts. If a school district charged with violations of the commission's regulations challenges the legislation creating the commission, is the court likely to uphold the legislation creating the commission? (A) No, because Congress is prohibited form delegating legislative powers to the executive branch. (B) No, because Congress is prohibited from delegating judicial powers to the executive branch. (C) Yes, because Congress is authorized to enact legislation to prohibit violations of Fourteenth Amendment rights. (D) Yes, because Congress is authorized to enact legislation that is necessary and proper to promote the general welfare.

(C). Congress has the authority to enforce civil rights pursuant to the Fourteenth Amendment Enabling Clause. The study of school district sports programs conducted by Congress showed discrimination based on race or national origin in violation of nonwhite students' equal protection rights. Because the state-run sports programs have been shown to interfere with students' Fourteenth Amendment rights, Congress has the authority to legislate to eliminate the restrictions placed on the student's exercise of those rights. The authority given to the vice president to review commission decisions does not infringe upon the judicial power of the federal courts. There is no violation of the constitutional separation of powers because the vice president is not the final arbiter of commisison decisions. While those decisions are appealed to the vice president, cases are "subject to judicial review by the federal courts." Thus, there is no violation of the federal courts' Article III powers.

QID: 9128 Defendant believed, in accordance with his religion, that only God can heal. When Defendant's son was hit by a car while playing outside his house, Defendant carried Son inside and began to pray fervently, but Son's condition only worsened. When Defendant's sister (a medical doctor) learned that Son was vomiting blood, she petitioned a court to have Son hospitalized. Her petition was granted, and Son was hospitalized. However, due to the delay in treatment, Son fell into a coma. Defendant then filed a motion to remove Son from the life-support systems, asserting that such artificial life-sustaining measures were not permitted in his faith. In the hearing on Defendant's motion, Son's doctors testified that Son was unlikely to ever regain consciousness. They further testified that the delay caused by Defendant was the reason that Son was now in this irreversible coma and that if the Son had received prompt treatment, he would likely have been released from the hospital in good condition the very next day. The court allowed the disconnection of Son's life-support systems, following which he died immediately. Of which, if any, of the following crimes is Defendant likely to be convicted in relation to Son's death? (A) Murder. (B) Voluntary manslaughter (C) Involuntary manslaughter (D) No crime

(C). Defendant had a duty to protect Son, so his failure to seek medical attention for Son most certainly makes him criminally liable for Son's death. Less clear, however, is whether Defendant's inaction constitutes gross negligence (making the crime in question involuntary manslaughter) or a "wanton and willful disregard" for Son's life (which would elevate the crime to depraved-heart murder). Given Defendant's devout faith and concern for his son's well-being, it seems unlikely that Defendant will be considered to have demonstrated a "wanton and willful disregard" for Son's life. The more likely outcome is a finding that gross negligence resulted in Son's death. As such, the crime of which Defendant is most likely to be convicted is involuntary manslaughter.

QID: 9191 A train car carrying explosives entered a railroad station at 6:15 pm. The manager of the station had left at 6:00 pm and was nowhere to be found. The conductor decided to leave the train and its contents in the train yard just past the station. The owner of the explosives came to the train yard to pick up his packages. Upon leaving the yard, he got into a car accident when another car ran a red light. The explosives rolled into the street, causing oncoming traffic to swerve. The plaintiff was driving home from work, saw the packages in the road, and swerved into a tree. The plaintiff is suing the owner of the explosives in strict liability for the damage to his car. Will the plaintiff prevail? (A) Yes, because carrying explosives is an abnormally dangerous activity. (B) Yes, because the damage to his car was caused by his swerving to avoid the explosives. (C) No, because the explosives did not explode before the plaintiff hit the tree. (D) No, because the owner of the explosives exercised due care while driving.

(C). If the defendant's abnormally dangerous activity leads to injury, the defendant will be strictly liable. While the transportation of explosives is abnormally dangerous, it is their explosive nature that creates the danger. The fact that the box of explosives rolled into the street, causing oncoming traffic to swerve to avoid hitting the boxes, has nothing to do with the abnormal dangerousness of explosives and will not cause the owner of the explosives to be strictly liable. If the explosives had exploded and caused the plaintiff to swerve and hit the tree, the owner of the explosives would have been strictly liable.

QID: 77266 A new anchor sued a cable television company under a state breach of contract claim. The anchor was a citizen of California, while the company was incorporated in Delaware and had its primary place of business in Georgia. The anchor claimed $200,000 in damages. The contract the anchor's claim was based on had a forum selection clause stating that any claim regarding the contract would be heard in California state court. The company was later successful in having the case removed to federal district court based on diversity jurisdiction. If the anchor believes the forum selection clause is enforceable, what would be his best option? (A) Make a motion for removal to the state judge assigned to hear the action. (B) Make a motion for removal to the federal judge assigned to hear the action. (C) Make a motion for remand to the federal judge assigned to hear the action. (D) Appeal the decision to the fedral appeals court.

(C). If the plaintiff believes that an action was removed to federal court improperly, he or she may make a motion for remand to the federal judge assigned to hear the action. Generally, appeals are allowed only on final judgments regarding the merits of the entire action. Here, the action has not yet received a final judgment.

QID: 101190 A movie director, who is a New York resident, has contracted with an actor to buy the actor's Oceanside estate in California and made a substantial down payment. When it is time for the closing on the estate, the actor states that he has been cast in a new movie that will pay him an enormous amount of money, and he has decided that he can easily afford to keep the estate. He tenders the down payment back to the director. The director files suit in federal court seeking specific performance of the sale contract and requesting a jury trial in his complaint. Is the director's complaint proper? (A) Yes, because the parties are residents of different states and the amount in controversy is surely over $75,000. (B) Yes, because a party can request a jury trial in a pleading that is served on other parties and filed with the court. (C) No, because a specific performance is an action in equity. (D) No, because a jury demand cannot be served in a complaint.

(C). In general, the director's complaint is proper. It appears to be subject to federal court jurisdiction, a party can make a jury trial request in a complaint and specific performance is an appropriate cause of action for a real estate sale contract. However, there is no right to a jury trial in an equitable action. Because an action for specific performance is in equity, there is no right to a jury.

QID: 77287 A construction company built an apartment building that collapsed and killed 10 people. The company had only $1 million in assets. The estates of the 10 people killed told the company that they planned to bring 10 separate actions, asking for damages of over $4 million in total. May the company bring an interpleader action to resolve its liability among the plaintiffs? (A) Yes, because the plaintiffs' claims far exceed the available assets. (B) Yes, because the plaintiff's claims all come from the same transaction or occurrence. (C) No, because interpleader is inappropriate in this action. (D) No, because an interpleader action can only be brought by a neutral third party.

(C). Interpleader is used where a plaintiff has some holding that would expose the plaintiff to multiple liability from adverse claims. The stakeholding party, or "stakeholder," can commence an action for interpleader to resolve liability where there are two or more adverse claimants. However, interpleader is only applicable where multiple claims demand the same thing or obligation -- usually a piece of property, price, or insurance policy. Here, while the company may only have $1 million in assets, its liability is legally unlimited and interpleader is inappropriate.

QID: 9192 A devoted daughter had not heard from her elderly father for several days. Concerned, the daughter decided to check on him. She entered her father's house and found him on the kitchen floor, unconscious, with a large bruise on his temple. She called 911, and the father was transported to the hospital for treatment. As she was leaving her father's house, the daughter noticed that the television and DVD player that she had given her father for Christmas were missing. The daughter contacted the police and reported the theft of the items. Two days later, police officers came to a suspect's home with a warrant for his arrest on drug charges. When the police entered the suspect's apartment, they immediately saw a television and DVD player in the middle of the living room. The television and DVD player were identified as belonging to the father, and the police arrested the suspect, who was subsequently charged with burglary and battery. At trial, the suspect claimed that he had found the television and DVD player in a trash can in an alley. Because the father had not seen his attacker's face, he was unable to identify the suspect as his attacker, and there was no physical evidence connecting the suspect to the incident. The suspect was acquitted of the burglary and battery charges. A short time later, the father, who had never fully recovered from the attack, died form a blood clot caused by the injury, and the suspect was charged with felony murder. At trial, the suspect moved for dismissal of the case on the ground that the felony murder charge violated the Double Jeopardy Clause. Should the suspect's motion be granted? (A) No, because the suspect had not yet been convicted of any charges related to the burglary of the father's home. (B) No, because the father's death had not yet occurred at the time that the suspect was tried on the burglary charges. (C) Yes, because the suspect was acquitted of the burglary charge. (D) Yes, because burglary and felony murder do not constitute the same offense.

(C). The Fifth Amendment prohibition on double jeopardy forbids a defendant from being tried twice for the same crime arising from the same set of facts. It incorporates the concept of collateral estoppel--a defendant may not be tried for a different crime arising out of the same criminal conduct if a previous prosecution necessarily determined factual issues required for conviction in the defendant's favor. For instance, for a defendant to be guilty of felony murder, the defendant must be found guilty of the underlying felony. Thus, in this case, for the suspect to be convicted of felony murder relating to a burglary, the prosecution would need to re-litigate facts pertaining to the underlying burglary charge. Given that these facts that had already been determined in the suspect's favor during his trial for burglary and battery, the felony murder charge violates the prohibition against double jeopardy and should be dismissed.

QID: 9174 The leader of a religious group that believes that dancing is sinful instructs his followers to kidnap and bind the feet of the lead dance of a prominent dance troupe. Two members of the religious group accost the dancer after one of his performance, tie him up, bind his feet, and leave him under a tree in the city park. The dancer is found almost immediately. He is unharmed, apart from a few bruises he acquired while fighting off his assailants. The leader of the religious group and the two members are charged with conspiracy kidnapping, assault, and battery. In the ensuing trial, which of the following statements is most accurate? (A) The court may not convict the defendants for acting on their religious beliefs regarding dancing. (B) The court may not determine whether the defendants were required to bind the dancer's feet based on their religious beliefs. (C) The court may not inquire into the reasonableness of the defendants' religious beliefs. (D) The court may not inquire into the sincerity of the defendants' religious beliefs.

(C). The Free Exercise Clause of the First Amendment prohibits the government from interfering with the freedom to choose any form of religion. Because the freedom to choose is absolute, the court cannot inquire into the reasonableness (i.e., truthfulness) of a particular religious belief. The court can, however, inquire into the sincerity of these beliefs, and it may convict a person for committing an illegal act in furtherance of his or her beliefs. The court is not prohibited from inquiring into the sincerity of religious beliefs. The court is not prohibited from inquiring into the sincerity of religious belief; it is prohibited only from inquiring into the reasonableness (i.e. truthfulness) of a particular religious belief.

QID: 9151 Corporations in a state began relocating their headquarters to other states. In an effort to stop the exodus, the state legislature enacted a bill requiring the state government to give preference to corporations located within the state in its purchasing decisions for goods and services purchased by state governmental entities. The governor signed the bill into law. A number of corporations that had been denied the opportunity to supply goods and services to the state government sued the state to enjoin the operation of the law. Assuming proper standing, how should the court rule? (A) In favor of the plaintiff corporations, because the law violates the Privileges and Immunities Clause of Article IV, Section 2 of the U.S. Constitution. (B) In favor of the plaintiff corporations, because the law violates the Privileges and Immunities Clause of the Fourteenth Amendment to the U.S. Constitution. (C) Against the plaintiff corporations, because the Privileges and Immunities Clause of Article IV, Section 2 of the U.S. Constitution does not apply to corporations. (D) Against the plaintiff corporations, because the Privileges and Immunities Clause of the Fourteenth Amendment to the U.S. Constitution does not apply to corporations.

(C). The Privileges and Immunities Clause of Article IV, Section 2 of the U.S. Constitution, provides: "The citizens of each state shall be entitled to all privileges and immunities of citizens in the several states." This clause prohibits states from discriminating against nonresidents with respect to "essential activities" or "fundamental rights." However, corporations and aliens are not "citizens" for purposes of the Article IV Privileges and Immunities Clause.

QID: 10362 A concerned citizen approached a federal agent about her friend's husband. She worried that her friend's husband might have child pornography in his possession. The federal agent asked the concerned citizen to keep an eye open next time she was at her friend's house and look around for any evidence. The next time the concerned citizen visited her friend, she asked the friend if she could use the friend's computer to check e-mail while the friend was cooking dinner. The friend agreed, and the concerned citizen discovered evidence of child pornography in a "secret" file on the computer. The citizen reported her findings to the federal agent, who obtained a warrant and seized the computer. The concerned citizen's friend and husband were charged with possession of child pornography, although the charges were later dropped against the friend. At trial, if the defendant objects, will the evidence form the computer found by the concerned citizen be admissible? (A) Yes, because the evidence was found by a private citizen. (B) Yes, because the concerned citizen had permission to use the computer. (C) No, because the search that produced the evidence was illegal. (D) No, because it is "fruit of poisonous tree."

(C). The concerned citizen was acting under the direction of a government agent, and therefore, her search of her friend's computer was governed by the Fourth Amendment. The concerned citizen obtained permission to use the computer to check her email. When the concerned citizen's friend agreed, it was for that limited use. Consent as a defense to a charge of illegal search has three elements: (1) the consent must be unforced and the result of an informed decision; (2) the search must not exceed the scope of the consent; and (3) the consenting party must have authority to consent; and (3) the consenting party must have authority to consent. In this case, the friend had no reason to know that a search was going to be conducted, so her consent was by trick. Furthermore, the consent to use the computer was for the concerned citizen to check email. The citizen's search for secret files exceeded the scope of her permission to use the computer. As such, the search was illegal, and the evidence will be suppressed.

QID: 9094 A fisherman runs a commercial boatyard in which boat owners store their vessels over the winter. The fisherman sells the boatyard to a retired navy admiral, conveying title by general warranty deed. The retired admiral operates the boatyard for a period of five years, then sells the property to a cruise ship captain, conveying title by quitclaim deed. The cruise ship captain operates the boatyard for a period of one year, at which time he is notified that a grantor prior to the fisherman retains an interest in the warehouse on the grounds for the boatyard and now demands that cruise ship captain vacate the structure. The cruise ship captain is forced to find alternate dry-dock facilities for several yachts housed in the warehouse over the winter. The cruise ship captain confront s the retired admiral and the fisherman about the claimant's interest, but both insist they have no knowledge of any prior claims or encumbrances. One-quarter of the property the cruise ship captain purchased is now unavailable to him, having been taken over by the claimant. If the cruise ship captain files an action seeking damages against the fisherman and the retired admiral, what is the court most likely to award the cruise ship captain? (A) Damages against the retired admiral, because the claimant's ejectment of the cruise ship captain. (B) Damages against the fisherman and the retired admiral, because the claimant's ejectment of the cruise ship captain breached the covenant of quiet enjoyment in the fisherman's deed to the retired admiral and the succeeding deed from the retired admiral to the cruise ship captain. (C) Damages against the fisherman, because the claimant's ejectment of the cruise ship captain breached the covenant of quiet enjoyment in the fisherman's deed to the retired admiral. (D) No damages, because knowledge of the claimant's interest in the boatyard cannot be imputed to the fisherman or the retired admiral.

(C). The fisherman conveyed the property to the retired navy man by general warranty deed, which contains the standard covenants of title: the covenant of seisin, of right to convey, against encumbrances, of quiet enjoyment, and of warranty. The covenant of quiet enjoyment warrants that the grantee's possession and enjoyment of the property will not be disturbed by any third party asserting a valid claim to the property. Here, the claimant has disturbed the cruise ship captain's possession and enjoyment of the property by ejecting him from the boatyard's warehouse. The covenant of quiet enjoyment runs with the land; that is, a remote grantee such as the cruise ship captain can sue the original grantor (the fisherman) for damages if a third-party claimant disturbs the grantee's possession and enjoyment. Thus, the court will likely award damages for the fisherman's breach of the covenant. Sweet Cats Envied Quiet Earthworms With Floppy Asparagus

QID: 28812 A homeowner hired a contractor to dig a well. They agreed that the job was to be completed within two weeks, for a price of $3,000. After digging for one week, the contractor hit solid rock that was impenetrable with his current drill bit. The occurrence of hitting solid rock was rare in this area, and had never happened to the contractor in the thirty previous wells he had dug in the neighborhood. He asked the homeowner for an additional $1000 to complete the job, which was fair and reasoanble in light of the circumstances. The homeowner agreed to the contractor's request. After ther job was completed, the contractor sent a bill to the homeowner for $4000, but the homeowner refused to pay. At modern law, how much is the homeowner required to pay? (A) $3,000, because there was no new consideration. (B) $3000, because the contractor was a professional. (C) $4000 because the modification was fair and reasonable. (D) $4000, because the contractor acted in good faith.

(C). The homeowner will be required to pay $4000 because of the unforeseen difficulties that arose during the course of performance. Modernly, a promise is enforceable, despite the preexisting duty rule, where circumstances arise not reasonably anticipated by the parties at the time of contracting. Here, the contractor's prior experience in the area, coupled with the rarity of such an occurrence in this locale, make it clear that the modification was fair and reasonable under these circumstances, and therefore the homeowner will be required to pay the modified price. Generally, new consideration is needed for a modification; however, the "unforeseen difficulties" doctrine permits an existing contract to be modified to account for unforeseen difficulties that arise during the course of performance. A promise to increase compensation under an existing contract is enforceable as a mutual modification under these facts.

QID: 10386 A mail carrier in a small town saved up for a down payment on a house. The mail carrier had never owned his own home before and was inexperienced with mortgages. When he and his wife found the home of their dreams, they made an offer to buy the house, which was accepted. The sale was contingent on the mail carrier securing a mortgage. Without much delay, the mail carrier found a company that agreed to carry his mortgage for the dream home. Among the various provisions in the mortgage note were: the loan amount and the interest rate; the term of the mortgage (30 years); a clause providing that prepayment would be subject to penalty; and an acceleration clause providing that, in the event of default, the mortgagee could declare the entire balance due and that the mortgagor would forfeit his right of redemption in the event that a foreclosure sale was consummated. Several years later, the mail carrier was laid off form his job and could not make his mortgage payments. He defaulted on the loan and the mortgagee began proceedings for a judicial foreclosure. The mail carrier moved with his wife into his parents' home and rented the mortgaged house to a busienssman who was in town for a few months. The mail carrier did not make any further payments on the defaulted loan. The foreclosure progressed, and eventually the mortgage company held a foreclosure sale. Two months after the foreclosure sale, the mail carrier inherited some money from his grandmother. The mail carrier went to redeem his dream home but was prevented by the terms of his contract form redeeming theh ome after the foreclosure sale. This jurisdiction recognizes both equitable and statutory rights of redemption. If the mail carrier brings an action to recover his home, what is the likely outcome? (A) THe mail carrier will not prevail, because he failed to redeem the home with the rent he received from the busienssman. (B) The mail carrier will not prevail, because a foreclosure sale terminates the mortgagor's interest in the mortgaged real estate. (C) The mail carrier will prevail, because the right of redemption may not be waived in the original purchase document. (D) The mail carrier will prevail because redemption cannot be waived on the purchase of a primary residence.

(C). The mortgagor cannot waive the right of redemption when the mortgage is created. Therefore, the clause purporting to waive the mail carrier's right of redemption after a foreclosure sale is ineffective. In some states, his right to redeem would have expired with the foreclosure sale anyway, but the question indicates that this jurisdiction recognizes the statutory right of redemption, therefore the mortgagor's right of redemption continues after the foreclosure sale (by matching the foreclosure price) for 6-12 months. The mail carrier tried to redeem his property two months after the sale. Although the mail carrier might have been able to redeem the property with the businessman's rent, he wasn't obligated to do so. The mail carrier could choose to allow the foreclosure sale to proceed. The mortgagee would probably bring an action against the mail carrier in an attempt to make up for any shortfall in the foreclosure sale and the rent money that the miail carrier collected would probably have to be used at that time to pay off his debt. But redemption is optional so this answer choice is incorrect. Note that this is a majority jurisdiction (lien theory), so it was within the mail carrier's right to collect rent from the property prior to the foreclosure.

QID: 9090 Two men have been successful bank robbers for many years. They convince their neighbor to join them in their most recent venture, which they spend several weeks planning. On the night before the robbery, the three men check their weapons, gloves, and masks. However, the neighbor gets cold feet and decides not to participate in the robbery, which he communicates to the two robbers. The neighbor then takes off his gloves and mask, leaves his gun, and walks home. The next night, the two robbers enter the bank and commit the robbery. As the robbers are getting into their van, they see a market patrol unit pull up alongside them. The robbers fired shots at the patrol officer. A bullet from one robber's gun strikes the officer in the head, killing him instantly. The robbers drive away, but soon encounter a roadblock and are captured by the police. As a result of the police officer's death, the police come down extremely hard on the robbers. They seize evidence with improper searches, obtain incriminating statements from them by threatening and brutalizing them, and conduct illegally suggestive identification procedures. All of this evidence is thrown out due to constitutional violations. With so much evidence ruled inadmissible, the district attorney is desperate to be done with the case. When the robbers offer to testify against a third actor, the district attorney offers them deals that include much shorter sentences than the crime would usually warrant. Using this testimony from the robbers, the district attorney prosecutes the neighbor. Assuming the robbers testify truthfully, what is the likely outcome of the neighbor's trial? (A) The neighbor will be found guilty of murder because it was foreseeable that someone could get killed during an armed robbery. (B) The neighbor will be found guilty of felony murder because the police officer was killed during the commission of a felony. (C) The neighbor will be found guilty of conspiracy because he agreed to rob the bank. (D) The neighbor will not be found guilty of any crime because he did not participate in the events that led to the killing of the police officer.

(C). The neighbor attempted to withdraw from the conspiracy before the robbery by clearly communicating his intention to withdraw and by leaving his mask, gloves, and gun and walking home. Under the common law rule, withdrawal is not a valid defense to a charge of conspiracy because the crime is committed as soon as the parties agreed to commit the crime. Withdrawal is, however, a defense to crimes committed in furtherance of the conspiracy after the defendant has withdrawn from the conspiracy. Thus, the neighbor will not be liable for either the robbery or the officer's death. Even under the Model Penal Code, which does recognize withdrawal as a defense to a charge of conspiracy, the neighbor did not effectively withdraw from the conspiracy because, while he did give timely notice to all members of the conspiracy of his intent to withdraw, the MPC also requires the withdrawing party to perform an affirmative act to thwart the conspiracy. In this case, the other two conspirators were accomplished bank robbers, and so merely leaving the scene was not an affirmative act that would thwart the success of the conspiracy. Thus, the neighbor can still be found guilty of conspiracy.

QID: 101178 The defendant served the plaintiff with a notice to take the deposition of a witness in a federal case. The notice stated that the deposition would be taken in ten days. Because of the short notice, the plaintiff's lawyer was unable to attend the deposition. The plaintiff's lawyer took no action and did not attend the deposition. The defendant filed a motion for summary judgment and attached portions of the transcript of the deposition of the witness in support of the motion. The plaintiff objected to the use of the transcript of the deposition against the plaintiff. Should the court rule that the deposition of the witness cannot be used against the plaintiff in connection with the defendant's motion for summary judgment? (A) Yes, because the plaintiff received less than 14 days notice of the deposition. (B) Yes, because depositions are not proper summary judgment evidence. (C) No, because plaintiff did not file a motion for protective order before the deposition occurred. (D) No, because the plaintiff did not obtain a ruling on a motion for protective order before the deposition took place.

(C). Under FRCP 32(a)(5)(A), a deposition may not be used against a party who received less than 14 days notice of the deposition if that party filed a motion for protective order that was pending at the time of the deposition. Here, the plaintiff received less than fourteen days' notice but took no action. By not filing a motion for protective order, the plaintiff lost the opportunity to preclude use of the deposition against the plaintiff despite the short notice.

QID: 9273 An athletic trainer owned a building containing an indoor sports center complete with a training room, basketball and tennis courts, and an indoor track. After a hurricane damaged the roof, the trainer observed the saturated walls of the training room and bemoaned the resulting water damage. A patron overheard him and said, "if you want to get rid of the place, just ask." The trainer hestitated a moment and then said, "Sure, why not. How about $750,000?" The patron handed the trainer a check for $7,500 as a down payment, and the parties agreed to meet the following morning to tender performance. That night, a tornado struck, tearing the entire roof off the sports center. The next day, the trainer appeared at the appointed time to tender the deed to the building. However, the patron refused to tender the purchase price, claiming he had not agreed to purchase "a dump with no roof." IF the trainer files suit against the patron for specific performance, who will prevail? (A) The trainer, under the doctrine of part performance. (B) The trainer, under the doctrine of equitable conversion. (C) The patron, because the agreement was not in writing. (D) The patron, because the trainer bore the risk of loss.

(C). Under the Statute of Frauds, a contract for the purchase or sale of real property must be in writing, containing all necessary provisions (including the price of the property and an adequate description), and signed by the party to be charged (that is, the defendant). Here, the agreement between the athletic trainer and the patron was not written. As such, the agreement was unenforceable, and the trainer cannot prevail in his action for specific performance.

QID: 9240 One morning while a grandmother was driving to visit her grandchildren, a motorist ran a stop sign and hit her car broadside. The grandmother sustained severe trauma to her head and was in the hospital for several weeks. Eventually, she was well enough to go home. However, the trauma to her head caused an injury to her inner ear. As a result, the grandmother occasssionally had trouble with her balance. One day, while the grandmother was at home, she had a sudden attack of vertigo, lost her balance, and fell. Unbeknownst to the motorist, the grandmother had osteoporosis, which made her bones very fragile. As a result, when she fell, she broke her hip. The grandmother sued the motorist for damages. For which of her physical injuries will the grandmother be able to recover from the motorist? (A) The injury to her head only, because the fall was an intervening force. (B) The injury to her head only, because the severity of the harm caused by the fall was unforeseeable. (C) The injuries to her head and to her hip, because the motorist's negligent conduct was the cause in fact of the grandmother's injuries. (D) The injuries to her head and to her hip, under the doctrine of res ipsa loquitur.

(C). Under the so-called "thin-skulled" or "eggshell" plaintiff rule, a defendant is liable for the full consequences of a plaintiff's injury even though, due to the plaintiff's peculiar susceptibility to harm (of which the defenant was unaware), those consequences were more severe than they would have been in a normal person. In this case, but for the motorist's negligence, the grandmother's injuries would not have occurred. Thus, the motorist's negligent conduct was the cause in fact of the grandmother's injuries. Because, under the circumstances, the doctrine of proximate cause provided no basis for limiting the motorist's liability, he is liable for all of the grandmother's injuries.

QID: 9103 A longtime resident of a close-knit community told another resident that a local businessman never gave anyone a fair deal. The businessman sued the resident for defamation. Which of the following facts would serve to strengthen the businessman's case against the resident. (A) The resident's statement was made orally and was never put in writing. (B) The businessman is a television reproter who covers restaurants and the arts. (C) The resident elaborated that he was referring to the businessman's tendency to cheat during neighborhood poker games. (D) The resident was in a deposition on an unrelated matter when he made the statement.

(C). Where the plaintiff in a defamation action is a private party and the subject matter is not a matter of public concern, the plaintiff need not prove actual malice. In contrast, a public figure plaintiff must prove that the defendant acted with malice (known falsity or recklessness as to truth or falsity). In this case, if the resident elaborated that he was referring to the businessman's tendency to cheat during neighborhood poker games, it would appear that the resident's comment related to a private matter, rather than a public matter, so the businessman would not be required to prove malice. Therefore, this fact would tend to support the businessman's defamation claim.

QID: 9197 Seller sold Blackacre, a parcel of land in a race-notice jurisdiction, to Buyer. To finance the purchase, Buyer took out a mortgage on Blackacre from Lender. Lender immediately recorded the mortgage. Buyer did not immediately record the deed. Seller then mortgaged Blackacre to Bank, which immediately recorded that mortgage. Finally, Buyer recorded his deed. Seller defaulted on the mortgage, and Bank sought to foreclose. Lender and Buyer objected to the foreclosure on the grounds that the Bank's mortgage was not valid. For whom should the court rule, and why? (A) Buyer and Lender, because they are bona fide purchasers. (B) Buyer and Lender, because Lender recorded before Bank. (C) Bank, because it has no notice of the conveyance to Buyer. (D) Bank, because it gave value for its interest.

(C). A race-notice recording statute protects subsequent bona fide purchasers for value without notice of who recorded first. For purposes of the recording statutes, mortgages are considered "purchasers" because they gave value for their interest. In this case, Bank is a bona fide purchaser who recorded before Buyer. Bank gave value for its interest. Bank also took without notice--neither a search of the grantor-grantee index nor a search of the grantee-grantor index would have disclosed the deed to Buyer. Thus, the mortgage in favor of Lender was outside the chain of title. As a bona fide purchaser without notice, Bank will be successful. The fact that Lender recorded first will not protect its interest. Lender's interest derives form Buyer's. Because Buyer failed to record his deed before Bank, Lender's deed is outside the chain of title available to Bank to search. Therefore, Bank could not have notice of the conveyance to Buyer or any of his successors in interest.

QID: 101180 A realtor working for an international realty company believed that he had not received the same bonuses and work conditions as other realtors in other countries because of his US nationality. He filed an employment discrimination claim in federal court. After the parties had made their initial disclosures and done some discovery, the realty company filed a motion for summary judgment. The realtor's response stated that his pleadings raised a sufficient question of material fact and that the court should not grant the motion. The court granted the motion. Did the court err? (A) Yes, because the parties had already completed some discovery. (B) Yes, because the court should have waited until discovery was complete. (C) No, because a summary judgment motion should be viewed in the light most favorable to the moving party. (D) No, because a response to summary judgment cannot rely on the pleadings to raise a material fact.

(D). A court considering a summary judgment motion should view the evidence in the light most favorable to the non-moving party, which would give the realtor the benefit of the doubt here. However, it is not enough for the non-moving party simply to rely on his pleadings in response to a summary judgment claim. The realtor's doing so meant that the court could rightly grant summary judgment. Under the Federal Rules, a party can bring a summary jdugment motion at any time until 30 days after discovery has closed. There is no need to wait until discovery is complete.

QID: 9168 Will owned a large ranch in fee simple absolute. He had always intended that his son and daughter, Jon and Willa, would take over the ranch when he could no longer operate it alone. To that end, Will had his lawyer prepare a deed conveying the ranch to Jon and Willa "as tenants in common." When Will presented his children with the deed, he asked them to give each other the right of first refusal before transferring either of their interests to a third party. Jon and Willa agreed, placing their hands on a Bible and solemnly swearing to honor Will's request. Jon and Willa ran the ranch together for five years without incident. However, Jon preferred technology to nature and eventually left to work in the city, giving Willa no indication of when, if ever, he would return. Willa continued to operate the ranch successfully without Jon's assistance and enjoyed substantial profits that she did not share with him. After four years of operating the ranch on her own, Willa decided that she could no longer handle the large property without help and conveyed her interest to her faithful ranch-hand, Becky. Jon was angry to learn of Willa's conveyance of her interest to Becky, and he sought a declaratory judgment deeming the conveyance invalid in light of Willa's agreement to give Jon the right of first refusal. What is the most likely outcome of Jon's lawsuit? (A) Jon will prevail, because Willa violated his co-tenancy rights. (B) Jon will prevail under an estoppel theory, if Becky knew about the right of first refusal agreement. (C) Jon will not prevail, because the right of first refusal agreement is an unenforceable restraint on alienation. (D) Jon will not prevail, because the right of first refusal agreement was unenforceable as violating the Statute of Frauds.

(D). A promise ot sell of transfer an interest in land must satisfy the Statute of Frauds requirements that the agreement be in writing, signed by the party to be charged, and contain the essential contract terms. A promise to give another the right of first refusal in an agreement concerning an interest in land that is within the Statute of Frauds. The oral promise by Jon and WIlla, even if sworn on a Bible, does not satisfy the statute's requirements. Therefore, Jon will fail in his effort to void Willa's conveyance of the ranch to Becky.

QID: 10405 A psychologist owned a small practice where he and two other psychologists counseled children. The business was near a spreading industrial area with very lax zoning laws. A number of adult entertainment businesses moved in nearby and the psychologist decided that it was time to relocate the practice. He sold the building and property to an entrepreneur who planned to open a tattoo parlor. The entrepreneur did not know that the zoning laws in effect on his newly purchased property barred a tattoo business at that location, even though just down the block he would have been in a different, acceptable zone. Further, the entrepreneur discovered that the rear of the building was infested with termites, the neighbor's fence encroached on the property by six inches, and that same neighbor possessed an easement to use the driveway on the entrepreneur's property to reach the back of his lot. Feeling like he'd been tricked into buying the property, the entrepreneur sued the psychologist to rescind the contract claiming that the psychologist had not delivered marketable title. Does the entrepreneur have valid grounds for his claim that the title was not marketable? (A) Yes, because of the termite infestation and the zoning law. (B) Yes, because of the zoning law and the encroaching fence. (C) Yes, because of the neighbor's encroaching fence. (D) Yes, because of the neighbor's driveway easement.

(D). An easement that burdens a property, such as a right of way, is an encumbrance that renders title unmarketable. Therefore, this is the correct answer. The existence of the zoning law does not affect the marketability of title, although the marketability of title might have been affected if the psychologist had been in violation of a zoning law. The fence only encroaches a few inches and will not render title unmarketable. Neither will the termites or any other physical defect.

QID: 108554 An ice cream manufacturer and an ice cream shop entered into a contract whereby the shop agreed to buy all the ice cream it needed from the manufacturer, and the manufacturer agreed to sell as much ice cream as it wanted to the shop. After several weeks, the manufacturer received its first request for inventory from the shop, but refused to sell any ice cream. Instead, the manufacturer chose to sell to the shop's main competitor, who offered more money. The shop had to find another vendor for its inventory, which cost them an additional 20% of their business. This resulted in $10,000 in overall losses the shop would not have suffered had the manufacturer delivered as promised. Which of the following is most accurate? (A) The manufacturer is liable for the full value of the contract. (B) The manufacturer is liable for the difference between the cover and contract prices. (C) The manufacturer is not liable, because there was not a liquidated damages clause contained in the agreement. (D) The manufacturer is not liable, because the contract is not enforceable.

(D). An illusory contract contains a promise that is unenforceable due to indefiniteness or lack of mutuality, because only one side is bound to perform. When the provisions of the purported promise render the performance of the person who makes the promise optional or completely within his or her discretion, pleasure, and control, nothing absolute is promised; and the promise is said to be illusory. Here, this contract is illusory, because the shop is bound to buy all of their ice cream from the manufacturer, while the manufacturer is bound to nothing at all. This contract is therefore not enforceable.

QID: 9269 A businessman runs a gourmet pizza shop in his hometown that makes deliveries throughout the county in distinctive three-wheeled purple vehicles adapted from conventional cars. After three years, the businessman gets ready to expand the business into an adjacent county, which happens to be in the next state. He signs leases for space in those counties and buys four additional custom-made vehicles for deliveries. The adjacent county has a regulation prohibiting three-wheeled vehicles from all of its secondary roads and most of its main roads because of the narrowness and steepness of those roads and the increased danger of rollover with three-wheeled vehicles. In federal court in the adjacent county, the businessman challenges the county's regulations as applied to his delivery vehicles. The county moves to dismiss on the basis that the businessman lacks standing. How is the court likely to rule on the question of whether the businessman has standing? (A) The court will decline jurisdiction, because the businessman has suffered no injury giving him standing. (B) The court will decline jurisdiction, because there has been no violation of the businessman's civil rights. (C) The court will accept jurisdiction, because the case presents a federal question on interstate commerce. (D) The court will accept jurisdiction, because the delivery vehicles the businessman ordered are unique.

(D). Before the federal court will exercise jurisdiction, a plaintiff must demonstrate standing to bring the action in federal court. The plaintiff must allege that his rights under the Constitution or federal law were violated, causing him direct injury. Here, the businessman has suffered a direct economic injury because he has bought custom-made delivery vehicles he cannot use as a consequence of the neighboring county's ban of the vehicles. This answer correctly notes that the injury he suffered as a result of the neighboring county's regulation gives him standing to bring his claim.

QID: 10410 A state has enacted a statute that requires companies or individuals who grow produce outside of the state to obtain a permit to sell the produce within the state. The permit is very expensive. Some growers living in a sister state are upset about the permit and believe it violates the Commerce Clause. Which of the following is true regarding a challenge to the state's statute? (A) The sister state may sue the state without its consent, but the federal government must obtain the state's consent to sue the state. (B) The federal government may sue the state without its consent, but the sister state must obtain the state's consent to sue the state. (C) The federal government or the sister state may sue the state without its consent, and the U.S. Supreme Court has original and exclusive jurisdiction in both cases. (D) The federal government or the sister state may sue the state without its consent, and the U.S. Supreme Court has original jurisdiction in both cases.

(D). Either the federal government or a sister state may sue a state without its consent. When the federal government (or one of its agencies or instrumentalities) sues a state, the U.S. Supreme Court has original, but not exclusive, jurisdiction. However, when a state is sued by a sister state, the Supreme Court has both original and exclusive jurisdiction. You should note that the Eleventh Amendment prohibits citizens of one state form suing another state in federal court.

QID: 101184 A man was arrested and charged with forgery. Seeing an opportunity to make some quick money, the man's friend indicated that a local court clerk owed him a favor and that for $1000, the friend could make sure that the clerk sent the man's case to the most lenient judge in the county, who also owed the friend a favor. The man agreed and paid the friend the $1000. It was later revealed that the friend never actually intended to talk to the clerk or judge. The man's case did eventually end up in front of the most lenient judge in the county. However, the friend never actually spoke to the clerk or the judge. The prosecution's main witness in the case failed to appear, and as a result, the judge dismissed the case for lack of prosecution. If the friend is prosecuted for theft of the $1000, what is the likely outcome? (A) The friend is not guilty because he never spoke to the clerk or the judge. (B) The friend is not guilty because the man received the desired outcome for the $1000. (C) The friend is guilty of larceny by trick because the friend never spoke to the clerk or the judge. (D) The friend is guilty of false pretenses because the friend never spoke to the clerk or the judge.

(D). False pretenses requires a misrepresentation of fact used to obtain title to money or property. Here, the friend was aware of the man's situation and procured title to the man's $1000 through a material misrepresentation. (C) is wrong because, while larceny by trick also requires a misrepresentation title does not pass in that situation. Here, title to the money passed from the man to the friend, and as a result, the proper crime is false pretenses rather than larceny by trick.

QID: 77289 A woman, a citizen of Ohio, was involved in a car accident in Idaho involving two other people. One of the other drivers, a citizen of Idaho, filed a lawsuit against the woman alleging $30,000 in damages, while the other driver, a citizen of Michigan, filed a lawsuit against the woman alleging $50,000 in damages. The plaintiffs wish to bring their action in federal court. Does the federal court have jurisdiction? (A) Yes, because the federal court has diversity jurisdiction over the matter. (B) Yes, because the federal court has federal question jurisdiction over the matter. (C) No, because the accident occurred in Idaho. (D) No, because the amount in controvery is insufficient.

(D). For diversity jurisdiciton the amount in controversy must be more than $75,000. In order to meet the $75,000 threshold for diversity jurisdiction, a claimant may combine each amount in controversy against a single defendant. However, more than one claimant cannot add their claims together. Here, there are two separate claimants, each with claims less than $75,000. If one of them had made a claim over $75,000, there might have been subject matter jurisdiciton.

QID: 9141 A nursing student has a live-in boyfriend, who has been arrested several times for violent crimes. He has never been convicted, however. The boyfriend has been physically abusive to the nursing student throughout their relationship, and she would like to leave him. However, he has indicated repeatedly that if she did so, he would hunt her down and kill her. Recently, the boyfriend has suspected that he is under surveillance by law enforcement, and his illegal business activities have been curtailed. However, he is getting desperate for money, so he tells the nursing student that she has to do a job for him or he will kill her slowly and painfully. If the nursing student commits the crime as her boyfriend demands, she is likely to be (A) acquitted (B) acquitted, if she honestly believed that the boyfriend would kill her if she did not commit the crime. (C) convicted. (D) Convicted, if she committed murder

(D). If the nursing student committed any crime other than murder, she might successfully raise the affirmative defense of duress/coercion. However, duress/coercion is not a defense to murder, because it would be equivalent to valuing one life over another.

QID: 10358 A scientist was on vacation in a rural part of the state where many peple practice a complex religion indigenous to the area that invovles various spells and necromancy. The scientist did not believe in nor practice the religion but was in the area to study bird calls. While following the sound of a bird she did not recognize, the scientist entered the property of a dealer. The dealer, an ex-convict who still dealt drugs from his remote property, practiced the local religion. When the scientist happened upon the dealer, the dealer was completing a drug sale. The buyer ran off, but the dealer decided to make sure that the scientist would not tell anybody about what she'd seen. The scientist assured the dealer that she meant no harm and was simply lost, but the dealer grabbed a vial out of his pocket that appeared to contain blood and telling the scientist that the liquid would cause her to lose her memory, flung the contents of the vial at the scientist. The scientist was merely grossed out because she did not know that the liquid contained an acid that could cause painful burns. The liquid came close but missed the scientist, who then turned and fled. Can the dealer be charged with criminal assault? (A) No, because the scientist was trespassing. (B) No, unless the state is a minority state defining assault as fear of battery. (C) Yes, but only if the state is a majority state defining assault as attempted battery. (D) Yes, whether the state is in the minority or the majority in defining assault.

(D). In the majority of states, the definition of criminal assault is attempted battery. The defendant must have intended to commit a battery for an assault charge to stick. In majority states, the fact that the victim was unaware of the danger does not defeat teh charge of assault because the focus is on the intent of the perpetrator. So the fact that the scientist was unaware that the liquid could burn her does not negate the assault charge. In minority states, the definition of criminal assault is fear of battery. In other words, if the victim is put in fear of immediate battery, then criminal assault may be charged. The threatened contact need only be offensive or insulting in this type of criminal assault. The victim does not have to fear actual injury or pain. So again, since the scientist was aware that some sort of bloody potion was being flung her way, her fear of getting covered with this offensive liquid gives rise to a charge of asasult, even though she didn't know the liquid could actually harm her. The elements of criminal assault exist whether it is a majority or miniority state and therefore, this is the correct answer choice.

QID: 28813 At his son's graduation party attended by dozens of classmates, friends, and family, a proud father toasted his son's accomplishment and announced that since his son had graduated high school, it was time to graduate to a new car as well. He went on to say, "Son, my baby is now your baby," pointing to his prized 1957 Chevy sitting int he driveway. Later that night, after one too many drinks, the father missed the top step and fell down the staircase. He was rushed to the hospital and pronounced dead. The son has requested the administrator of his father's estate honor his father's wishes and gives him the 1957 Chevy. The administrator, who was not at the party, has refused the son's request. At trial, to determine the ownership of the 1957 Chevy, the son wants to call a friend to testify as to his father's statement regarding the car. The administrator of the father's estate objects. How should the court rule? (A) The evidence is inadmissible, because it is hearsay not within an exception. (B) The evidence is inadmissible, because of the dead man's statute. (C) The evidence is admissible because it is a dying declaration. (D) The evidence is admissible because it has independent legal significance.

(D). Statements constituting verbal acts or legally operative acts are not hearsay, because they are not offered for their truth. Rather, certain words have independent legal significance, such as words of a contract offer or acceptance, libel, slander, threats, etc. Other examples include words that have the effect of making a gift, such as is the case here. The law attaches rights and obligations to certain words simply because they are said. Here, the father's statement would be admissible in order for the son to attempt to prove that he received the car as a gift, and that, therefore, the car would not be a part of the father's estate upon his death.

QID: 101173 A woman in Mexico with diabetes took a medication that was made by a United States manufacturer at a factory in Mexico. After several years, she discovered that it had worsened her condition and that, as a result, she would need a kidney transplant. She filed suit in the federal district court in the United States where the manufacturer was headquartered. The manufacturer believed that it would be difficult to defend the suit in the court where it was filed because the doctors involved, information about manufacturing processes, and other information related ot the suit were in Mexico. If the manufacturer files a motion for forum non conveniens, seeking transfer of the case to Mexico, should the court grant it? (A) Yes, because U.S. court should not be burdened with a dispute that has its nexus in another county. (B) Yes, because locating the case in Mexico would decrease the costs and amount of time of trial. (C) No, because federal courts should grant deference to the plaintiff's choice of forum. (D) No, because federal courts cannot transfer venue to a foreign country.

(D). The doctrine of forum non conveniens does apply when the most convenient court is a foreign one. It is also the case that deference to a plaintiff's choice of forum is lessened if the plaintiff is a foreign citizen. The problem here is that the court would have no power to transfer venue outside of the U.S. As a result, the court can only dismiss the case and leave the plaintiff to refile elsewhere.

QID: 28822 A basketball fan had an opportunity to attend the championship game of a national tournament. He had attended many sporting events in his community, but never one this large and important. He arrived early to the arena and was shown to his seat. Spotting his favorite player warming up on the court, the fan left his seat and went court side to try to get an autograph. The fan ducked past the security guard and quickly walked onto the court, where he tripped over a piece of the flooring. The flooring in that area of the court had not been properly secured after the maintenance crew replaced a large section of the floor the previous night. The fan fractured his wrist as a result of the fall. If the fan brings suit against the arena for his injuries, will he prevail. (A) Yes, because he was an invitee. (B) Yes, because the arena did not properly rope off the court. (C) No, because he assumed the risk of injury. (D) No, because he was a trespasser.

(D). The fan was a paying guest and was therefore an invitee. Invitees are owed the duty to inspect and make safe. His injury was due to a dangerous condition on the land (the unsecured flooring). However, once the fan ducked by the security guard and then stepped onto the court, he became a trespasser and therefore will not recover for his injuries.

QID: 10384 An opera singer contracted with the owner of a barn to purchase the barn and the two acres of land surrounding it. The opera singer planned to convert the barn into a recording studio. Before the deed was signed and the property paid for a creek near the barn flooded the property and caused considerable damage to the barn, which the opera singer had already begun renovating. The opera singer decided to cancel the contract and refused to pay. If the seller sues the opera singer for specific performance in a jurisdiction adhering to the Uniform Vendor and Purchaser Risk Act, who will prevail? (A) The opera singer, because specific performance is not available as a seller's remedy. (B) The opera singer, because risk of loss remained with the seller until the same was completed. (C) The seller, because the barn was not completely destroyed. (D) The seller, because risk of loss had transferred to the opera singer.

(D). The majority of states still follow the common law rule of equitable conversion. Under the equitable conversion rule, once a binding contract for the sale of real estate has been executed, then the risk of loss is on the buyer. Where states have adopted the Uniform Vendor and Purchaser Risk Act, the risk of loss remains with the seller during that period after the contract was executed but before the deed has been conveyed. However, even in a state that has adopted the Uniform Vendor and Purchaser Risk Act, the risk of loss transfers to the purchaser once the purchaser takes possession or takes legal title. Here, the opera singer had begun renovating the barn. This indicates that she had taken possession and therefore the risk of loss was hers. The seller will be entitled to specific performance.

QID: 9143 When Husband lost his job, his marriage became very strained. Eventually he moved out and began living at a nearby hotel, but neither spouse filed for divorce. Husband saw Wife at the local convenience store. He took her aside and whispered, "Get out of here. Don't tell anyone, but something is going to go down and I do not want you to be here." Wife left immediately without saying a word, only to discover the next day that the convenience store had been robbed. Husband was arrested for the robbery. The prosecution wants Wife to testify as to what Husband told her in the store. Can Husband prevent Wife from testifying about his statement? (A) No, because Wife is free to testify, but may choose not to. (B) Yes, because her testimony is hearsay. (C) Yes, because under the spousal testimony privilege, one spouse can prevent the other from testifying. (D) Yes, because under the spousal communication privilege, one spouse can prevent the other form testifying.

(D). The spousal communication privilege extends to confidential communications made during a valid marriage. The privilege applies here because the statement appears to be confidential and was made during the marriage (Despite the fact that Husband has moved out, the facts clearly state that there has not yet been a divorce. Because both spouses must waive the privilege for a valid waiver to occur, Husband will be able to prevent the Wife from testifying as to his statement. Note that Husband's statement is not "in furtherance of an ongoing crime," because Husband appears to be warning Wife, not furthering the crime; there is no joint participation apparent from these facts. The spousal testimony privilege, sometimes referred to as spousal immunity, prevents one spouse from being compelled to testify against the other spouse in a criminal proceeding, where the mariage is valid at the time of the testimony. The privilege is held by the testifying spouse (Wife). Therefore, Husband may not use this privilege to prevent Wife from testifying as to his statement in the store.

QIDL 9259 The Gingko Hill Times' business section offers financial "makeovers" for average people in need of money management and investment advice. Each week, a fincncial adviser hired by the newspaper meets with an individual, couple, or family whose portfolio of assets, debts, and investments is chronicled int he "makeover" article, and the advisor provides specific and detailed financial advice. While some of the advisers work for private firms, some are newspaper employees. The most popular adviser is Marlon, a business writer for the newspaper, and his financial question-and-answer column appears three times each week. Marlon writes the "makeover" article on the first Monday of each month and is paid an additional fee for that work. A state statute requires that all persons "holding themselves out as financial advisers and providing services for a fee shall be licensed by the state to so provide financial advice." If Marlon is charged with violating the statute and raises a free speech defense, who is likely to prevail? (A) The state, because there is a rational relation between regulating the commercial speech of financial advisers and the legitimate government interest in protecting advisee's assets. (B) The state, because commercial speech receives a lower level of protection that other types of speech. (C) Marlon, because there is no substantial government interest in regulating the content of his speech. (D) Marlon, because there is no compelling state interest in regulating the content of his speech.

(D). The statute in question regulates the subject matter of Marlon's speech by prohibiting him from addressing financial issues without a license. Content-based regulation of protected speech is subject to strict scrutiny, so the state must show a compelling government interest in the regulation and that it does not have less restrictive means to this end. (Regulations affecting unprotected speech, such as obscenity, defamation, or commercial speech, are scrutinized under a lesser standard.) Here, because the statute regulates Marlon's speech based on content, it will be subject to strict scrutiny. As such, this answer is correct.

QID: 9157 An entrepreneur bough a vacant lot on which he decided to build a new office building. He contracted with a developer to complete the office building and deliver the deed by the inside date of October 1, with an outside date of November 1. By November 2, neither the developer nor the entrepreneur had tendered performance. On November 4, the entrepreneur e-mailed the developer that he was canceling the contract because the developer had not finished or delivered title by November 1. The developer replied by e-mail that he was entitled to a reasonable time to finish because excess rain had delayed construction. The developer also indicated that he would be ready to close by November 24. By November 24, the developer had obtained a certificate of occupancy from the city and the appropriate title documents. He tendered his performance, but the entrepreneur refused to accept the developer's performance. The developer sued the entrepreneur for breach of contract. On which of the following factors is the court most likely to base its decision? (A) Whether the entrepreneur's efforts to terminate the contract were in good faith. (B) Whether the entrepreneur can show that the delay caused him undue hardship. (C) Whether the developer exercised due diligence in completing the office building (D) Whether time was of the essence, as demonstrated by the use of an inside date and an outside date.

(D). Where a specific time is set for performance of a contract, this provision will be enforced. Where no specific time limit is included in a contract, courts will apply a "reasonable" time standard. Thus, if parties to a contract agree that time is of the essence, their contract should include a provision to that effect. Here, the inclusion in the contract of such terms as an "inside date" and an "outside date" indicates that time was of the essence in the construction of the office building. As such, the entrepreneur would have the right to cancel the contract when the developer failed to complete construction and to deliver title within the time frame specified in the contract. While it is true that the developer may attempt to show that he was duly diligent in constructing the office building and obtaining title and that unforeseeable weather problems unavoidably delayed construction, the critical factor in this case is whether time was of the essence in the contract. If a court finds that the parties agreed that time was of the essence, then the developer's failure to render performance within the specified time frame would constitute a breach of an essential term of the contract, thereby entitling the entrepreneur to cancel it -- regardless of the developer's good faith or due diligence.


Kaugnay na mga set ng pag-aaral

Great Depression unit test study guide

View Set

Chapter 3: The Role of Evidence in Criminal Investigations

View Set

Ms. Robles-Suffix (end of words)---

View Set

Marketing Management Chapter Ten

View Set